CINXE.COM
irregularity - What is the reason for having irregular verbs? - Linguistics Stack Exchange
<!DOCTYPE html> <html itemscope itemtype="https://schema.org/QAPage" class="html__responsive " lang="en"> <head> <title>irregularity - What is the reason for having irregular verbs? - Linguistics Stack Exchange</title> <link rel="shortcut icon" href="https://cdn.sstatic.net/Sites/linguistics/Img/favicon.ico?v=9bd85c4a2e0d"> <link rel="apple-touch-icon" href="https://cdn.sstatic.net/Sites/linguistics/Img/apple-touch-icon.png?v=e73d3c3a0e44"> <link rel="image_src" href="https://cdn.sstatic.net/Sites/linguistics/Img/apple-touch-icon.png?v=e73d3c3a0e44"> <link rel="search" type="application/opensearchdescription+xml" title="Linguistics Stack Exchange" href="/opensearch.xml"> <link rel="canonical" href="https://linguistics.stackexchange.com/questions/37503/what-is-the-reason-for-having-irregular-verbs" /> <meta name="viewport" content="width=device-width, height=device-height, initial-scale=1.0, minimum-scale=1.0"> <meta name="bingbot" content="noarchive"> <meta property="og:type" content= "website" /> <meta property="og:url" content="https://linguistics.stackexchange.com/questions/37503/what-is-the-reason-for-having-irregular-verbs"/> <meta property="og:site_name" content="Linguistics Stack Exchange" /> <meta property="og:image" itemprop="image primaryImageOfPage" content="https://cdn.sstatic.net/Sites/linguistics/Img/apple-touch-icon@2.png?v=20cdfff28de0" /> <meta name="twitter:card" content="summary"/> <meta name="twitter:domain" content="linguistics.stackexchange.com"/> <meta name="twitter:title" property="og:title" itemprop="name" content="What is the reason for having irregular verbs?" /> <meta name="twitter:description" property="og:description" itemprop="description" content="Having irregular verbs makes the language more complex. Users have to memorize more rules.
Is there a historical reason, or some other reason, that English had all these irregular verbs?" /> <script id="webpack-public-path" type="text/uri-list">https://cdn.sstatic.net/</script> <script src="https://ajax.googleapis.com/ajax/libs/jquery/3.7.1/jquery.min.js"></script> <script defer src="https://cdn.sstatic.net/Js/third-party/npm/@stackoverflow/stacks/dist/js/stacks.min.js?v=90a447add688"></script> <script src="https://cdn.sstatic.net/Js/stub.en.js?v=30464efc364f"></script> <link rel="stylesheet" type="text/css" href="https://cdn.sstatic.net/Shared/stacks.css?v=bd2aba62c5a4"> <link rel="stylesheet" type="text/css" href="https://cdn.sstatic.net/Sites/linguistics/primary.css?v=e192c5b2b284"> <link rel="alternate" type="application/atom+xml" title="Feed for question 'What is the reason for having irregular verbs?'" href="/feeds/question/37503"> <script> StackExchange.ready(function () { StackExchange.using("postValidation", function () { StackExchange.postValidation.initOnBlurAndSubmit($('#post-form'), 2, 'answer'); }); StackExchange.question.init({showAnswerHelp:true,totalCommentCount:3,shownCommentCount:3,enableTables:true,questionId:37503}); styleCode(); StackExchange.realtime.subscribeToQuestion('312', '37503'); StackExchange.using("gps", function () { StackExchange.gps.trackOutboundClicks('#content', '.js-post-body'); }); }); </script> <link rel="stylesheet" type="text/css" href="https://cdn.sstatic.net/Shared/Channels/channels.css?v=5981bb1a5bd7"> <script> StackExchange.ready(function () { StackExchange.realtime.init('wss://qa.sockets.stackexchange.com'); StackExchange.realtime.subscribeToReputationNotifications('312'); StackExchange.realtime.subscribeToTopBarNotifications('312'); }); </script> <script type="application/json" data-role="module-args" data-module-name="Shared/options.mod">{"options":{"locale":"en","serverTime":1739733043,"routeName":"Questions/Show","stackAuthUrl":"https://stackauth.com","networkMetaHostname":"meta.stackexchange.com","site":{"name":"Linguistics Stack Exchange","description":"Q\u0026A for professional linguists and others with an interest in linguistic research and theory","isNoticesTabEnabled":true,"enableNewTagCreationWarning":false,"insertSpaceAfterNameTabCompletion":false,"id":312,"cookieDomain":".stackexchange.com","childUrl":"https://linguistics.meta.stackexchange.com","negativeVoteScoreFloor":null,"enableSocialMediaInSharePopup":true,"protocol":"https"},"user":{"fkey":"b39cb1a761f93912de068f8e6267ee5702b7d597a570755c5112354b099addd4","tid":"8f2d1d36-cffc-4c49-848c-f5ef926ed59d","rep":0,"isAnonymous":true,"isAnonymousNetworkWide":true},"realtime":{"newest":true,"active":true,"tagged":true,"staleDisconnectIntervalInHours":0},"events":{"postType":{"question":1},"postEditionSection":{"title":1,"body":2,"tags":3}}}}</script> <script type="application/json" data-role="module-args" data-module-name="Shared/settings.mod">{"settings":{"auth":{"oauthInPopup":true},"accounts":{"currentPasswordRequiredForChangingStackIdPassword":true},"search":{},"legal":{"oneTrustTCFConfigId":"cb0f3c87-b769-4e66-bbaa-377f9194216d","useCustomConsent":false},"site":{"stacksEditorPreviewEnabled":true,"forceHttpsImages":true,"enableImageHttps":true,"enableUserHovercards":true,"allowImageUploads":true},"image":{"maxImageUploadSizeInBytes":10485760,"maxImageUploadSizeInBytesAnimatedGif":2097152},"subscriptions":{"defaultFreemiumMaxTrueUpSeats":50,"defaultMaxTrueUpSeats":1000,"defaultBasicMaxTrueUpSeats":250},"markdown":{"enableTables":true},"userMessaging":{},"flags":{"allowRetractingCommentFlags":true,"allowRetractingFlags":true},"paths":{"jQueryUIJSPath":"https://ajax.googleapis.com/ajax/libs/jqueryui/1.12.0/jquery-ui.min.js","jQueryUICSSPath":"https://ajax.googleapis.com/ajax/libs/jqueryui/1.12.0/themes/smoothness/jquery-ui.css"},"questions":{"enableQuestionTitleLengthLiveWarning":true,"questionTitleLengthStartLiveWarningChars":50,"enableSavesFeature":true,"maxTitleSize":150},"snippets":{"renderDomain":"stacksnippets.net"},"comments":{},"questionLinkTitleReplacement":{"maxNumberOfSitesProcessed":10,"maxReplacementsPerSite":20},"intercom":{"appId":"inf0secd"},"elections":{"opaVoteResultsBaseUrl":"https://www.opavote.com/results/"},"tags":{},"mentions":{"maxNumUsersInDropdown":50}}}</script> <script>StackExchange.init();</script> <script> StackExchange.using.setCacheBreakers({"Js/adops.en.js":"6da43f5e0a84","Js/ask.en.js":"","Js/begin-edit-event.en.js":"20edbaccceae","Js/copy-transpiled.en.js":"f48f8f039185","Js/events.en.js":"","Js/explore-qlist.en.js":"ee2a4f8c3992","Js/full-anon.en.js":"c5f441a0c0f3","Js/full.en.js":"e5895fea2680","Js/highlightjs-loader.en.js":"a3e36fe85053","Js/inline-tag-editing.en.js":"8517756a2cb6","Js/keyboard-shortcuts.en.js":"c255a5a5979b","Js/markdown-it-loader.en.js":"781aa77e9602","Js/mentions-transpiled.en.js":"bcadb48d3570","Js/moderator.en.js":"c12dd4738c7f","Js/postCollections-transpiled.en.js":"9f7a87e8cf0d","Js/post-validation.en.js":"44d59ccfc490","Js/question-editor.en.js":"","Js/review-v2-transpiled.en.js":"c9e05881f8ab","Js/revisions.en.js":"9dd135bb585f","Js/stacks-editor.en.js":"cf77c9d2187c","Js/tageditor.en.js":"5b8f289d02ef","Js/tageditornew.en.js":"ca426894aed0","Js/tagsuggestions.en.js":"762b96a1ae1f","Js/unlimited-transpiled.en.js":"8713a979101d","Js/wmd.en.js":"eef1c2cfa021"}); StackExchange.using("gps", function() { StackExchange.gps.init(false); }); </script> <noscript id="noscript-css"><style>body,.s-topbar{margin-top:1.9em}</style></noscript> </head> <body class="question-page unified-theme"> <div id="signup-modal-container"></div> <script type="application/json" data-role="module-args" data-module-name="islands/signup-modal/index.mod">{"ContainerElementId":"signup-modal-container","FKey":"b39cb1a761f93912de068f8e6267ee5702b7d597a570755c5112354b099addd4","TriggerEvent":"signupModalShow","OauthInPopup":true,"ReturnUrl":"https://linguistics.stackexchange.com/questions/37503/what-is-the-reason-for-having-irregular-verbs","ReturnUrlForPopup":"https://linguistics.stackexchange.com/users/after-signup/oauth-only","SiteName":"Linguistics","SiteLogoPath":"https://cdn.sstatic.net/Sites/linguistics/Img/icon-48.png?v=df49e0cbe297","AuthProviders":["Google"],"ParentSiteUrl":"","IsInitiallyVisible":false}</script> <script defer src="https://cdn.sstatic.net/Js/webpack-chunks/svelte.en.js?v=ac1681a633a2"></script><script defer src="https://cdn.sstatic.net/Js/webpack-chunks/stacks-svelte.en.js?v=585200d05e44"></script><script defer src="https://cdn.sstatic.net/Js/webpack-chunks/8901.en.js?v=9ff34567d3a5"></script><script defer src="https://cdn.sstatic.net/Js/islands/signup-modal.en.js?v=47ec828efb46"></script> <script defer> dispatchEvent(new CustomEvent("openSignupModal")); </script> <div id="one-tap-container"></div> <script type="application/json" data-role="module-args" data-module-name="islands/one-tap/index.mod">{"ContainerElementId":"one-tap-container","FKey":"b39cb1a761f93912de068f8e6267ee5702b7d597a570755c5112354b099addd4","GoogleClientId":"717762328687-iludtf96g1hinl76e4lc1b9a82g457nn.apps.googleusercontent.com","Autoselect":false,"ReturnUrl":"https%3a%2f%2flinguistics.stackexchange.com%2fquestions%2f37503%2fwhat-is-the-reason-for-having-irregular-verbs"}</script><script defer src="https://cdn.sstatic.net/Js/webpack-chunks/svelte.en.js?v=ac1681a633a2"></script><script defer src="https://cdn.sstatic.net/Js/islands/one-tap.en.js?v=bc67e8c0f01e"></script> <div id="notify-container"></div> <div id="custom-header"></div> <header class="s-topbar ps-fixed t0 l0 js-top-bar"> <a href="#content" class="s-topbar--skip-link">Skip to main content</a> <div class="s-topbar--container"> <a href="#" class="s-topbar--menu-btn js-left-sidebar-toggle" role="menuitem" aria-haspopup="true" aria-controls="left-sidebar" aria-expanded="false"><span></span></a> <div class="topbar-dialog leftnav-dialog js-leftnav-dialog dno"> <div class="left-sidebar js-unpinned-left-sidebar" data-can-be="left-sidebar" data-is-here-when="sm"></div> </div> <a href="#" class="s-topbar--logo network-logo js-gps-track js-network-logo" data-gps-track="stack_exchange_popup.show" role="menuitem" aria-haspopup="true" aria-controls="topbar-network-logo-dialog" aria-expanded="false"> <svg aria-hidden="true" class="native mtn1 svg-icon iconSEAlternativeSm" width="107" height="15" viewBox="0 0 107 15"><path fill="#FEFEFE" d="m48.41 11.93-1.96-3.2-1.04 1.16v2.04h-1.42V2.18h1.42v6.01L48.14 5h1.72l-2.44 2.7 2.74 4.22zm-7.06.08c-1.59 0-3.14-.96-3.14-3.56s1.55-3.54 3.14-3.54c.97 0 1.65.27 2.31.97l-.97.93c-.44-.48-.79-.66-1.34-.66q-.84.01-1.3.62c-.31.38-.42.87-.42 1.68s.1 1.32.41 1.7c.3.4.76.62 1.3.62.56 0 .9-.18 1.35-.66l.97.92c-.66.7-1.34.98-2.31.98m-5.66-3.15h-1.65c-.83 0-1.26.37-1.26 1s.4.99 1.3.99c.53 0 .93-.04 1.3-.4q.32-.29.31-1.03zm.03 3.07v-.63c-.51.5-1 .71-1.87.71s-1.46-.2-1.89-.63a2 2 0 0 1-.55-1.49c0-1.16.82-2 2.42-2h1.86v-.5c0-.87-.44-1.3-1.54-1.3-.77 0-1.15.18-1.54.68l-.92-.86c.66-.77 1.35-1 2.52-1q2.9 0 2.9 2.38v4.64zm-5.9 0c-1.32 0-1.93-.93-1.93-1.93V6.18h-.8V5.1h.8V3h1.41v2.1h1.36v1.07H29.3v3.75c0 .5.25.81.78.81h.58v1.2zm-6.33.08c-1.48 0-2.55-.34-3.49-1.28l1-.98c.72.72 1.51.94 2.52.94 1.3 0 2.04-.55 2.04-1.5q0-.65-.39-1.01c-.25-.23-.5-.33-1.08-.41l-1.16-.17a3.4 3.4 0 0 1-1.88-.78 2.4 2.4 0 0 1-.72-1.86c0-1.7 1.25-2.86 3.3-2.86 1.3 0 2.22.33 3.07 1.1l-.96.94a3 3 0 0 0-2.15-.75c-1.16 0-1.8.65-1.8 1.52q-.02.54.37.9c.25.22.65.38 1.11.45l1.13.17c.91.13 1.42.35 1.84.72.54.47.8 1.17.8 2 0 1.8-1.48 2.86-3.55 2.86"/><path fill="#2F96E8" d="M104.16 7.09c-.2-.42-.6-.74-1.2-.74s-.99.32-1.18.74c-.1.25-.15.44-.16.75h2.7a2 2 0 0 0-.16-.75m-2.54 1.96c0 .9.56 1.57 1.55 1.57.78 0 1.16-.21 1.61-.66l1.08 1.04a3.4 3.4 0 0 1-2.7 1.11c-1.68 0-3.29-.76-3.29-3.62 0-2.3 1.26-3.6 3.1-3.6 1.97 0 3.1 1.44 3.1 3.37v.79zm-5.48-2.57C95.1 6.48 95 7.37 95 8.3s.1 1.85 1.15 1.85 1.18-.91 1.18-1.85c0-.93-.13-1.82-1.18-1.82m-.17 8.22c-1.1 0-1.84-.21-2.58-.92l1.1-1.11c.4.38.8.54 1.4.54 1.06 0 1.43-.74 1.43-1.46v-.72c-.47.51-1 .7-1.7.7-.69 0-1.29-.23-1.68-.62-.67-.66-.73-1.57-.73-2.8 0-1.24.06-2.13.73-2.8.4-.39 1-.62 1.7-.62.75 0 1.24.2 1.73.75v-.67h1.72v6.8c0 1.7-1.21 2.93-3.12 2.93m-5.76-2.67V7.76c0-.96-.61-1.28-1.17-1.28s-1.18.32-1.18 1.28v4.27h-1.78V4.97h1.73v.65a2.4 2.4 0 0 1 1.78-.73q1.07.02 1.67.62c.58.57.73 1.24.73 2v4.52zm-7.1-2.98h-1.4c-.64 0-1 .3-1 .8 0 .49.33.81 1.02.81.5 0 .8-.04 1.12-.34q.28-.25.26-.89zm.04 2.98v-.6c-.48.47-.93.67-1.74.67q-1.2 0-1.82-.62c-.38-.4-.58-.97-.58-1.59 0-1.12.77-2.05 2.42-2.05h1.68V7.5c0-.77-.38-1.11-1.32-1.11-.68 0-1 .16-1.37.58l-1.13-1.1c.7-.75 1.38-.97 2.57-.97q3 0 3.02 2.5v4.64zm-6.93 0v-4.3c0-.94-.6-1.25-1.15-1.25-.56 0-1.15.32-1.15 1.24v4.31h-1.77V2.38h1.77v3.24a2.4 2.4 0 0 1 1.7-.73c1.56 0 2.38 1.08 2.38 2.57v4.57zm-6.96.08c-1.42 0-3.18-.76-3.18-3.62 0-2.85 1.76-3.6 3.18-3.6.98 0 1.72.3 2.34.95l-1.2 1.2c-.36-.4-.68-.56-1.14-.56q-.62-.01-1.01.46c-.27.33-.4.8-.4 1.55s.13 1.24.4 1.58q.39.46 1 .46c.47 0 .79-.16 1.15-.56l1.2 1.18c-.62.65-1.36.96-2.34.96m-5.53-.08-1.3-2.11-1.3 2.11H59l2.45-3.6-2.35-3.46h2.12L62.42 7l1.21-2.02h2.13L63.4 8.43l2.46 3.6zm-11.75 0V2.06h6.6V3.8h-4.65v2.33h3.96v1.74h-3.96v2.42h4.65v1.74z"/><path fill="#8FD8F7" d="M0 3c0-1.1.9-2 2-2h8a2 2 0 0 1 2 2z"/><path fill="#155397" d="M12 10H0c0 1.1.9 2 2 2h5v3l3-3a2 2 0 0 0 2-2"/><path fill="#46A2D9" d="M0 4h12v2H0z"/><path fill="#2D6DB5" d="M0 7h12v2H0z"/></svg> </a> <div class="topbar-dialog network-logo-dialog js-network-logo-dialog dno" id="topbar-network-logo-dialog" role="dialog" aria-labelledby="topbar-network-logo-dialog-title" aria-describedby="topbar-network-logo-dialog-body"> <div class="dialog-content"> <h4 class="bold" id="topbar-network-logo-dialog-title">Stack Exchange Network</h4> <p id="topbar-network-logo-dialog-body"> Stack Exchange network consists of 183 Q&A communities including <a href="https://stackoverflow.com">Stack Overflow</a>, the largest, most trusted online community for developers to learn, share their knowledge, and build their careers. </p> <a class="s-btn s-btn__filled" href="https://stackexchange.com" data-gps-track="stack_exchange_popup.click">Visit Stack Exchange</a> <button class="s-btn s-btn__muted p0 ps-absolute t16 r16 js-close-button" aria-label="Close"><svg aria-hidden="true" class="svg-icon iconClear" width="18" height="18" viewBox="0 0 18 18"><path d="M15 4.41 13.59 3 9 7.59 4.41 3 3 4.41 7.59 9 3 13.59 4.41 15 9 10.41 13.59 15 15 13.59 10.41 9z"/></svg></button> </div> </div> <form id="search" role="search" action=/search class="s-topbar--searchbar js-searchbar " autocomplete="off"> <div class="s-topbar--searchbar--input-group"> <input name="q" type="text" role="combobox" placeholder="Search on Linguistics…" value="" autocomplete="off" maxlength="240" class="s-input s-input__search js-search-field wmn1 " aria-label="Search" aria-controls="top-search" data-controller="s-popover" data-action="focus->s-popover#show" data-s-popover-placement="bottom-start" /> <svg aria-hidden="true" class="s-input-icon s-input-icon__search svg-icon iconSearch" width="18" height="18" viewBox="0 0 18 18"><path d="m18 16.5-5.14-5.18h-.35a7 7 0 1 0-1.19 1.19v.35L16.5 18zM12 7A5 5 0 1 1 2 7a5 5 0 0 1 10 0"/></svg> <div class="s-popover p0 wmx100 wmn4 sm:wmn-initial js-top-search-popover" id="top-search" role="menu"> <div class="s-popover--arrow"></div> <div class="s-popover--content"> <div class="js-spinner p24 d-flex ai-center jc-center d-none"> <div class="s-spinner s-spinner__sm fc-orange-400"> <div class="v-visible-sr">Loading…</div> </div> </div> <span class="v-visible-sr js-screen-reader-info"></span> <div class="js-ac-results overflow-y-auto hmx3 d-none"></div> <div class="js-search-hints" aria-describedby="Tips for searching"></div> </div> </div> </div> </form> <nav class="h100 ml-auto overflow-x-auto pr12" aria-label="Topbar"> <ol class="s-topbar--content" role="menubar"> <li role="none"> <a href="/help" class="s-topbar--item js-help-button" role="menuitem" title="Help Center and other resources" aria-haspopup="true" aria-controls="topbar-help-dialog" data-ga="["top navigation","help menu click",null,null,null]"><svg aria-hidden="true" class="svg-icon iconHelp" width="18" height="18" viewBox="0 0 18 18"><path d="M9 1C4.64 1 1 4.64 1 9s3.64 8 8 8 8-3.64 8-8-3.64-8-8-8m.81 12.13c-.02.71-.55 1.15-1.24 1.13-.66-.02-1.17-.49-1.15-1.2.02-.72.56-1.18 1.22-1.16.7.03 1.2.51 1.17 1.23M11.77 8c-.59.66-1.78 1.09-2.05 1.97a4 4 0 0 0-.09.75c0 .05-.03.16-.18.16H7.88c-.16 0-.18-.1-.18-.15.06-1.35.66-2.2 1.83-2.88.39-.29.7-.75.7-1.24.01-1.24-1.64-1.82-2.35-.72-.21.33-.18.73-.18 1.1H5.75c0-1.97 1.03-3.26 3.03-3.26 1.75 0 3.47.87 3.47 2.83 0 .57-.2 1.05-.48 1.44"/></svg></a> </li> <div class="topbar-dialog help-dialog js-help-dialog dno" id="topbar-help-dialog" role="menu"> <div class="modal-content"> <ul> <li> <a href="/tour" class="js-gps-track s-block-link" data-gps-track="help_popup.click({ item_type:1 })" data-ga="["top navigation","tour submenu click",null,null,null]"> Tour <span class="item-summary"> Start here for a quick overview of the site </span> </a> </li> <li> <a href="/help" class="js-gps-track s-block-link" data-gps-track="help_popup.click({ item_type:4 })" data-ga="["top navigation","help center",null,null,null]"> Help Center <span class="item-summary"> Detailed answers to any questions you might have </span> </a> </li> <li> <a href="https://linguistics.meta.stackexchange.com" class="js-gps-track s-block-link" data-gps-track="help_popup.click({ item_type:2 })" data-ga="["top navigation","meta submenu click",null,null,null]"> Meta <span class="item-summary"> Discuss the workings and policies of this site </span> </a> </li> <li> <a href="https://stackoverflow.co/" class="js-gps-track s-block-link" data-gps-track="help_popup.click({ item_type:6 })" data-ga="["top navigation","about us submenu click",null,null,null]"> About Us <span class="item-summary"> Learn more about Stack Overflow the company, and our products </span> </a> </li> </ul> </div> </div> <li role="none"> <a href="https://stackexchange.com" class="s-topbar--item js-site-switcher-button js-gps-track" data-gps-track="site_switcher.show" aria-label="Site switcher" role="menuitem" title="A list of all 183 Stack Exchange sites" aria-haspopup="true" aria-expanded="false" data-ga="["top navigation","stack exchange click",null,null,null]"> <svg aria-hidden="true" class="svg-icon iconStackExchange" width="18" height="18" viewBox="0 0 18 18"><path d="M15 1H3a2 2 0 0 0-2 2v2h16V3a2 2 0 0 0-2-2M1 13c0 1.1.9 2 2 2h8v3l3-3h1a2 2 0 0 0 2-2v-2H1zm16-7H1v4h16z"/></svg> </a> </li> <li class="js-topbar-dialog-corral" role="presentation"> <div class="topbar-dialog siteSwitcher-dialog dno" role="menu"> <div class="header fw-wrap"> <h3 class="flex--item"> <a href="https://linguistics.stackexchange.com">current community</a> </h3> <div class="flex--item fl1"> <div class="ai-center d-flex jc-end"> <button class="js-close-button s-btn s-btn__muted p0 ml8 d-none sm:d-block" type="button" aria-label="Close" > <svg aria-hidden="true" class="svg-icon iconClear" width="18" height="18" viewBox="0 0 18 18"><path d="M15 4.41 13.59 3 9 7.59 4.41 3 3 4.41 7.59 9 3 13.59 4.41 15 9 10.41 13.59 15 15 13.59 10.41 9z"/></svg> </button> </div> </div> </div> <div class="modal-content bg-blue-200 current-site-container"> <ul class="current-site"> <li class="d-flex"> <div class="fl1"> <a href="https://linguistics.stackexchange.com" class="current-site-link d-flex gx8 site-link js-gps-track" data-id="312" data-gps-track="site_switcher.click({ item_type:3 })"> <div class="favicon favicon-linguistics site-icon flex--item" title="Linguistics"></div> <span class="flex--item fl1"> Linguistics </span> </a> </div> <div class="related-links"> <a href="https://linguistics.stackexchange.com/help" class="js-gps-track" data-gps-track="site_switcher.click({ item_type:14 })">help</a> <a href="https://chat.stackexchange.com?tab=site&host=linguistics.stackexchange.com" class="js-gps-track" data-gps-track="site_switcher.click({ item_type:6 })">chat</a> </div> </li> <li class="related-site d-flex"> <div class="L-shaped-icon-container"> <span class="L-shaped-icon"></span> </div> <a href="https://linguistics.meta.stackexchange.com" class="s-block-link px16 d-flex gx8 site-link js-gps-track" data-id="314" data-gps-track="site.switch({ target_site:314, item_type:3 }),site_switcher.click({ item_type:4 })"> <div class="favicon favicon-linguisticsmeta site-icon flex--item" title="Linguistics Meta"></div> <span class="flex--item fl1"> Linguistics Meta </span> </a> </li> </ul> </div> <div class="header" id="your-communities-header"> <h3> your communities </h3> </div> <div class="modal-content" id="your-communities-section"> <div class="call-to-login"> <a href="https://linguistics.stackexchange.com/users/signup?ssrc=site_switcher&returnurl=https%3a%2f%2flinguistics.stackexchange.com%2fquestions%2f37503%2fwhat-is-the-reason-for-having-irregular-verbs" class="login-link js-gps-track" data-gps-track="site_switcher.click({ item_type:10 })">Sign up</a> or <a href="https://linguistics.stackexchange.com/users/login?ssrc=site_switcher&returnurl=https%3a%2f%2flinguistics.stackexchange.com%2fquestions%2f37503%2fwhat-is-the-reason-for-having-irregular-verbs" class="login-link js-gps-track" data-gps-track="site_switcher.click({ item_type:11 })">log in</a> to customize your list. </div> </div> <div class="header"> <h3><a href="https://stackexchange.com/sites">more stack exchange communities</a> </h3> <a href="https://stackoverflow.blog" class="float-right">company blog</a> </div> <div class="modal-content"> <div class="child-content"></div> </div> </div> </li> <li role="none"><button class="s-topbar--item s-btn s-btn__icon s-btn__muted d-none sm:d-inline-flex js-searchbar-trigger" role="menuitem" aria-label="Search" aria-haspopup="true" aria-controls="search" title="Click to show search"><svg aria-hidden="true" class="svg-icon iconSearch" width="18" height="18" viewBox="0 0 18 18"><path d="m18 16.5-5.14-5.18h-.35a7 7 0 1 0-1.19 1.19v.35L16.5 18zM12 7A5 5 0 1 1 2 7a5 5 0 0 1 10 0"/></svg></button></li> <li role="none"> <a href="https://linguistics.stackexchange.com/users/login?ssrc=head&returnurl=https%3a%2f%2flinguistics.stackexchange.com%2fquestions%2f37503%2fwhat-is-the-reason-for-having-irregular-verbs" class="s-topbar--item s-topbar--item__unset s-btn s-btn__outlined ws-nowrap js-gps-track" role="menuitem" rel="nofollow" data-gps-track="login.click" data-ga="["top navigation","login button click",null,null,null]">Log in</a> </li> <li role="none"><a href="https://linguistics.stackexchange.com/users/signup?ssrc=head&returnurl=https%3a%2f%2flinguistics.stackexchange.com%2fquestions%2f37503%2fwhat-is-the-reason-for-having-irregular-verbs" class="s-topbar--item s-topbar--item__unset ml4 s-btn s-btn__filled ws-nowrap js-signup-button js-gps-track" role="menuitem" rel="nofollow" data-gps-track="signup.topbar.click" data-ga="["sign up","Sign Up Navigation","Header",null,null]">Sign up</a></li> </ol> </nav> </div> </header> <script> StackExchange.ready(function () { StackExchange.topbar.init(); }); StackExchange.scrollPadding.setPaddingTop(50, 10); </script> <div id="announcement-banner" class="js-announcement-banner bg-black-500 fc-white baw0 ff-sans fs-body2 py2" data-campaign="2025-02-11.community-ama" data-cookie="notice-ama" data-expire-date="1740582000000"> <div class="d-flex jc-space-between ai-center wmx12 mx-auto px16 py8"> <div class="flex--item mr12 "> <span> Join Stack Overflow’s first live community AMA on February 26th, at 3 PM ET. </span> <a target="_blank" class="s-link s-link__underlined fw-bold fc-white js-link js-gps-track" href="https://meta.stackexchange.com/questions/406399/join-us-for-our-first-community-wide-ama-ask-me-anything-with-stack-overflow-s?utm_medium=ppc&utm_source=stackoverflow-community&utm_campaign=community-ama&utm_content=announcement-banner1" data-ga="["community-ama","Announcement Banner","https://meta.stackexchange.com/questions/406399/join-us-for-our-first-community-wide-ama-ask-me-anything-with-stack-overflow-s?utm_medium=ppc&utm_source=stackoverflow-community&utm_campaign=community-ama&utm_content=announcement-banner1",null,{"dimension4":"community-ama"}]" data-gps-track="announcement_banner.click({campaign: '2025-02-11.community-ama', location: 2, action: 2})">Learn more</a> </div> <a class="flex--item fc-white js-dismiss js-gps-track" href="#" title="dismiss" data-gps-track="announcement_banner.click({campaign: '2025-02-11.community-ama', location: 2, action: 1})"><svg aria-hidden="true" class="m0 svg-icon iconClear" width="18" height="18" viewBox="0 0 18 18"><path d="M15 4.41 13.59 3 9 7.59 4.41 3 3 4.41 7.59 9 3 13.59 4.41 15 9 10.41 13.59 15 15 13.59 10.41 9z"/></svg></a> </div> </div> <script> StackExchange.ready(function () { StackExchange.Notice.announcementBannerInit(); }); </script> <header class="site-header"> <div class="site-header--container"> <a class="site-header--link fs-headline1 fw-bold" href="https://linguistics.stackexchange.com"> Linguistics </a> </div> </header> <div class="container"> <div id="left-sidebar" data-is-here-when="md lg" class="left-sidebar js-pinned-left-sidebar ps-relative"> <div class="left-sidebar--sticky-container js-sticky-leftnav"> <nav aria-label="Primary"> <ol class="nav-links"> <li> <ol class="nav-links"> <li class="ps-relative" aria-current="false"> <a href="/" class="s-block-link pl8 js-gps-track nav-links--link -link__with-icon" data-gps-track="top_nav.click({is_current: false, location:2, destination:8, has_activity_notification:False});home_nav.click({location:2})" aria-controls="" data-controller=" " data-s-popover-placement="right" aria-current="false" data-s-popover-auto-show="true" data-s-popover-hide-on-outside-click="never" > <div class="d-flex ai-center"> <svg aria-hidden="true" class="svg-icon iconHome" width="18" height="18" viewBox="0 0 18 18"><path d="M15 10v5a2 2 0 0 1-2 2H5a2 2 0 0 1-2-2v-5H0l9-9 9 9zm-8 1v6h4v-6z"/></svg> <span class="-link--channel-name pl6">Home</span> </div> </a> </li> <li class="ps-relative youarehere" aria-current="true"> <a id="nav-questions" href="/questions" class="s-block-link pl8 js-gps-track nav-links--link -link__with-icon" data-gps-track="top_nav.click({is_current: true, location:2, destination:1, has_activity_notification:False})" aria-controls="" data-controller=" " data-s-popover-placement="right" aria-current="false" data-s-popover-auto-show="true" data-s-popover-hide-on-outside-click="never" > <div class="d-flex ai-center"> <svg aria-hidden="true" class="svg-icon iconQuestion" width="18" height="18" viewBox="0 0 18 18"><path d="m4 15-3 3V4c0-1.1.9-2 2-2h12c1.09 0 2 .91 2 2v9c0 1.09-.91 2-2 2zm7.75-3.97c.72-.83.98-1.86.98-2.94 0-1.65-.7-3.22-2.3-3.83a4.4 4.4 0 0 0-3.02 0 3.8 3.8 0 0 0-2.32 3.83q0 1.93 1.03 3a3.8 3.8 0 0 0 2.85 1.07q.94 0 1.71-.34.97.66 1.06.7.34.2.7.3l.59-1.13a5 5 0 0 1-1.28-.66m-1.27-.9a5 5 0 0 0-1.5-.8l-.45.9q.5.18.98.5-.3.1-.65.11-.92 0-1.52-.68c-.86-1-.86-3.12 0-4.11.8-.9 2.35-.9 3.15 0 .9 1.01.86 3.03-.01 4.08"/></svg> <span class="-link--channel-name pl6">Questions</span> </div> </a> </li> <li class="ps-relative" aria-current="false"> <a href="/tags" class="s-block-link pl8 js-gps-track nav-links--link -link__with-icon" data-gps-track="top_nav.click({is_current: false, location:2, destination:2, has_activity_notification:False})" aria-controls="" data-controller=" " data-s-popover-placement="right" aria-current="false" data-s-popover-auto-show="true" data-s-popover-hide-on-outside-click="never" > <div class="d-flex ai-center"> <svg aria-hidden="true" class="svg-icon iconTags" width="18" height="18" viewBox="0 0 18 18"><path d="M9.24 1a3 3 0 0 0-2.12.88l-5.7 5.7a2 2 0 0 0-.38 2.31 3 3 0 0 1 .67-1.01l6-6A3 3 0 0 1 9.83 2H14a3 3 0 0 1 .79.1A2 2 0 0 0 13 1z" opacity=".4"/><path d="M9.83 3a2 2 0 0 0-1.42.59l-6 6a2 2 0 0 0 0 2.82L6.6 16.6a2 2 0 0 0 2.82 0l6-6A2 2 0 0 0 16 9.17V5a2 2 0 0 0-2-2zM12 9a2 2 0 1 1 0-4 2 2 0 0 1 0 4"/></svg> <span class="-link--channel-name pl6">Tags</span> </div> </a> </li> <li class="pb24"></li> <li class="ps-relative" aria-current="false"> <a id="nav-users" href="/users" class="s-block-link pl8 js-gps-track nav-links--link -link__with-icon" data-gps-track="top_nav.click({is_current: false, location:2, destination:3, has_activity_notification:False})" aria-controls="" data-controller=" " data-s-popover-placement="right" aria-current="false" data-s-popover-auto-show="true" data-s-popover-hide-on-outside-click="never" > <div class="d-flex ai-center"> <svg aria-hidden="true" class="svg-icon iconPeople" width="18" height="18" viewBox="0 0 18 18"><path d="M17 14c0 .44-.45 1-1 1H9a1 1 0 0 1-1-1H2c-.54 0-1-.56-1-1 0-2.63 3-4 3-4s.23-.4 0-1c-.84-.62-1.06-.59-1-3s1.37-3 2.5-3 2.44.58 2.5 3-.16 2.38-1 3c-.23.59 0 1 0 1s1.55.71 2.42 2.09c.78-.72 1.58-1.1 1.58-1.1s.23-.4 0-1c-.84-.61-1.06-.58-1-3s1.37-3 2.5-3 2.44.59 2.5 3c.05 2.42-.16 2.39-1 3-.23.6 0 1 0 1s3 1.38 3 4"/></svg> <span class="-link--channel-name pl6">Users</span> </div> </a> </li> <li class="ps-relative" aria-current="false"> <a id="nav-unanswered" href="/unanswered" class="s-block-link pl8 js-gps-track nav-links--link -link__with-icon" data-gps-track="top_nav.click({is_current: false, location:2, destination:5, has_activity_notification:False})" aria-controls="" data-controller=" " data-s-popover-placement="right" aria-current="false" data-s-popover-auto-show="true" data-s-popover-hide-on-outside-click="never" > <div class="d-flex ai-center"> <svg aria-hidden="true" class="svg-icon iconAnswer" width="18" height="18" viewBox="0 0 18 18"><path d="M14 15H3c-1.09 0-2-.91-2-2V4c0-1.1.9-2 2-2h12c1.09 0 2 .91 2 2v14zm-1.02-3L9.82 4H8.14l-3.06 8h1.68l.65-1.79h3.15l.69 1.79zm-2.93-3.12H7.9l1.06-2.92z"/></svg> <span class="-link--channel-name pl6">Unanswered</span> </div> </a> </li> </ol> </li> <li class="js-freemium-cta ps-relative mt32 mb8"> <div class="fs-fine tt-uppercase fc-black-600 fw-bold ml8 mt16 mb8">Teams</div> <div class="px12 pt12 pb4 mb12 fc-medium overflow-hidden"> <img class="wmx100 mx-auto mb12 h-auto d-block" width="151" height="24" src="https://cdn.sstatic.net/Img/teams/teams-promo.svg?v=e507948b81bf" alt=""> <p class="fs-fine"> Ask questions, find answers and collaborate at work with Stack Overflow for Teams. </p> <a href="https://stackoverflowteams.com/teams/create/free/?utm_medium=referral&utm_source=linguistics-community&utm_campaign=side-bar&utm_content=explore-teams" class="w100 s-btn s-btn__filled s-btn__xs bg-orange-400 h:bg-orange-500 js-gps-track pt8 pr7 pb6 pl7" data-gps-track="teams.create.left-sidenav.click({ Action: 6 })" data-ga="["teams left navigation - anonymous","left nav free cta","stackoverflow.com/teams/create/free",null,null]">Try Teams for free</a> <a href="https://stackoverflow.co/teams/?utm_medium=referral&utm_source=linguistics-community&utm_campaign=side-bar&utm_content=explore-teams" class="w100 s-btn s-btn__muted s-btn__xs mt1 js-gps-track" data-gps-track="teams.create.left-sidenav.click({ Action: 5 })" data-ga="["teams left navigation - anonymous","left nav free cta","stackoverflow.com/teams",null,null]">Explore Teams</a> </div> </li> <li class="d-flex ai-center jc-space-between ml8 mt32 mb8 js-create-team-cta d-none"> <a href="javascript:void(0)" class="s-link d-flex fl-grow1 fc-black-400 h:fc-black-600 fs-fine js-gps-track" role="button" aria-controls="popover-teams-create-cta" data-controller="s-popover" data-action="s-popover#toggle" data-s-popover-placement="bottom-start" data-s-popover-toggle-class="is-selected" data-gps-track="teams.create.left-sidenav.click({ Action: ShowInfo })" data-ga="["teams left navigation - anonymous","left nav show teams info",null,null,null]" > <div class="flex--item fl-grow1 fc-black-600 fw-bold tt-uppercase">Teams</div> <div class="flex--item px12"> <svg aria-hidden="true" class="svg-icon iconPlusSm" width="14" height="14" viewBox="0 0 14 14"><path d="M8 2H6v4H2v2h4v4h2V8h4V6H8z"/></svg> </div> </a> </li> <li class="ps-relative js-create-team-cta d-none"> <p class="fs-fine pr8 pl8 pb4 fc-black-400"> Ask questions, find answers and collaborate at work with Stack Overflow for Teams. <a href="https://stackoverflow.co/teams/?utm_medium=referral&utm_source=linguistics-community&utm_campaign=side-bar&utm_content=explore-teams-compact" class="s-link s-link__grayscale s-link__underlined fw-bold">Explore Teams</a> </p> </li> </ol> </nav> </div> <div class="s-popover ws2" id="popover-teams-create-cta" role="menu" aria-hidden="true"> <div class="s-popover--arrow"></div> <div class="ps-relative overflow-hidden"> <p class="mb2"><strong>Teams</strong></p> <p class="mb12 fs-caption fc-black-400">Q&A for work</p> <p class="mb12 fs-caption fc-black-500">Connect and share knowledge within a single location that is structured and easy to search.</p> <a href="https://stackoverflow.co/teams/" class="js-gps-track s-btn s-btn__filled s-btn__xs" data-gps-track="teams.create.left-sidenav.click({ Action: CtaClick })" data-ga="["teams left navigation - anonymous","left nav cta","stackoverflow.com/teams",null,null]"> Learn more about Teams </a> </div> <div class="ps-absolute t8 r8"> <svg aria-hidden="true" class="fc-orange-400 svg-spot spotPeople" width="48" height="48" viewBox="0 0 48 48"><path d="M13.5 28a4.5 4.5 0 1 0 0-9 4.5 4.5 0 0 0 0 9M7 30a1 1 0 0 1 1-1h11a1 1 0 0 1 1 1v5h11v-5a1 1 0 0 1 1-1h12a1 1 0 0 1 1 1v10a2 2 0 0 1-2 2H33v5a1 1 0 0 1-1 1H20a1 1 0 0 1-1-1v-5H8a1 1 0 0 1-1-1zm25-6.5a4.5 4.5 0 1 0 9 0 4.5 4.5 0 0 0-9 0M24.5 34a4.5 4.5 0 1 0 0-9 4.5 4.5 0 0 0 0 9" opacity=".2"/><path d="M16.4 26.08A6 6 0 1 0 7.53 26C5.64 26.06 4 27.52 4 29.45V40a1 1 0 0 0 1 1h9a1 1 0 1 0 0-2h-4v-7a1 1 0 1 0-2 0v7H6v-9.55c0-.73.67-1.45 1.64-1.45H16a1 1 0 0 0 .4-1.92M12 18a4 4 0 1 1 0 8 4 4 0 0 1 0-8m16.47 14a6 6 0 1 0-8.94 0A3.6 3.6 0 0 0 16 35.5V46a1 1 0 0 0 1 1h14a1 1 0 0 0 1-1V35.5c0-1.94-1.64-3.42-3.53-3.5M20 28a4 4 0 1 1 8 0 4 4 0 0 1-8 0m-.3 6h8.6c1 0 1.7.75 1.7 1.5V45h-2v-7a1 1 0 1 0-2 0v7h-4v-7a1 1 0 1 0-2 0v7h-2v-9.5c0-.75.7-1.5 1.7-1.5M42 22c0 1.54-.58 2.94-1.53 4A3.5 3.5 0 0 1 44 29.45V40a1 1 0 0 1-1 1h-9a1 1 0 1 1 0-2h4v-7a1 1 0 1 1 2 0v7h2v-9.55A1.5 1.5 0 0 0 40.48 28H32a1 1 0 0 1-.4-1.92A6 6 0 1 1 42 22m-2 0a4 4 0 1 0-8 0 4 4 0 0 0 8 0"/><g opacity=".35"><path d="M17 10a1 1 0 011-1h12a1 1 0 110 2H18a1 1 0 01-1-1m1-5a1 1 0 100 2h12a1 1 0 100-2zM14 1a1 1 0 00-1 1v12a1 1 0 001 1h5.09l4.2 4.2a1 1 0 001.46-.04l3.7-4.16H34a1 1 0 001-1V2a1 1 0 00-1-1zm1 12V3h18v10h-5a1 1 0 00-.75.34l-3.3 3.7-3.74-3.75a1 1 0 00-.71-.29z"/></g></svg> </div> </div> </div> <div id="content" class=""> <div itemprop="mainEntity" itemscope itemtype="https://schema.org/Question"> <link itemprop="image" href="https://cdn.sstatic.net/Sites/linguistics/Img/apple-touch-icon.png?v=e73d3c3a0e44"> <div class="inner-content clearfix"> <div id="question-header" class="d-flex sm:fd-column"> <h1 itemprop="name" class="fs-headline1 ow-break-word mb8 flex--item fl1"><a href="/questions/37503/what-is-the-reason-for-having-irregular-verbs" class="question-hyperlink">What is the reason for having irregular verbs?</a></h1> <div class="ml12 aside-cta flex--item sm:ml0 sm:mb12 sm:order-first d-flex jc-end"> <div class="ml12 aside-cta flex--item print:d-none"> <a href="/questions/ask" class="ws-nowrap s-btn s-btn__filled"> Ask Question </a> </div> </div> </div> <div class="d-flex fw-wrap pb8 mb16 bb bc-black-200"> <div class="flex--item ws-nowrap mr16 mb8" title="2020-10-31 03:31:23Z"> <span class="fc-black-400 mr2">Asked</span> <time itemprop="dateCreated" datetime="2020-10-31T03:31:23">4 years, 3 months ago</time> </div> <div class="flex--item ws-nowrap mr16 mb8"> <span class="fc-black-400 mr2">Modified</span> <a href="?lastactivity" class="s-link s-link__inherit" title="2020-11-02 12:15:50Z">4 years, 3 months ago</a> </div> <div class="flex--item ws-nowrap mb8" title="Viewed 773 times"> <span class="fc-black-400 mr2">Viewed</span> 773 times </div> </div> <div id="mainbar" role="main" aria-label="question and answers"> <div class="question js-question" data-questionid="37503" data-position-on-page="0" data-score="4" id="question"> <style> </style> <div class="js-zone-container zone-container-main"> <div id="dfp-tlb" class="everyonelovesstackoverflow everyoneloves__top-leaderboard everyoneloves__leaderboard"></div> <div class="js-report-ad-button-container " style="width: 728px"></div> </div> <div class="post-layout "> <div class="votecell post-layout--left"> <div class="js-voting-container d-flex jc-center fd-column ai-center gs4 fc-black-300" data-post-id="37503" data-referrer="None"> <button class="js-vote-up-btn flex--item s-btn s-btn__muted s-btn__outlined bar-pill bc-black-225 f:bc-theme-secondary-400 f:bg-theme-secondary-400 f:fc-black-050 h:bg-theme-primary-200" id="upvote-btn-37503" data-controller="s-tooltip" data-s-tooltip-placement="right" title="This question shows research effort; it is useful and clear" aria-pressed="false" aria-label="Up vote" data-selected-classes="fc-theme-primary-100 bc-theme-primary-500 bg-theme-primary-500" data-unselected-classes="bc-black-225 f:bc-theme-secondary-400 f:bg-theme-secondary-400 f:fc-black-050 h:bg-theme-primary-200"> <svg aria-hidden="true" class="svg-icon iconArrowUp" width="18" height="18" viewBox="0 0 18 18"><path d="M1 12h16L9 4z"/></svg> </button> <input type="hidden" id="voteUpHash" value="67:3:31e,16:32661f1506c7f4cc,10:1739733043,16:acf105292023d5d3,5:37503,24c8d4fb84324ac50d5d410cf9416a0ccc69e2c82f794ea2e08bc8102778a2ef" /> <div class="js-vote-count flex--item d-flex fd-column ai-center fc-theme-body-font fw-bold fs-subheading py4" itemprop="upvoteCount" data-value="4"> 4 </div> <button class="js-vote-down-btn js-vote-down-question flex--item mb8 s-btn s-btn__muted s-btn__outlined bar-pill bc-black-225 f:bc-theme-secondary-400 f:bg-theme-secondary-400 f:fc-black-050 h:bg-theme-primary-200" title="This question does not show any research effort; it is unclear or not useful" aria-pressed="false" aria-label="Down vote" data-selected-classes="fc-theme-primary-100 bc-theme-primary-500 bg-theme-primary-500" data-unselected-classes="bc-black-225 f:bc-theme-secondary-400 f:bg-theme-secondary-400 f:fc-black-050 h:bg-theme-primary-200"> <svg aria-hidden="true" class="svg-icon iconArrowDown" width="18" height="18" viewBox="0 0 18 18"><path d="M1 6h16l-8 8z"/></svg> </button> <input type="hidden" id="voteDownHash" value="67:3:31e,16:9a0e296ac433ff47,10:1739733043,16:a212ff30f098681c,5:37503,37cb3ce1afdd00ff7574857a49e6439c120499f0843b71a4af97e6a272e75b24" /> <button class="js-saves-btn s-btn s-btn__unset c-pointer py4" type="button" id="saves-btn-37503" data-controller="s-tooltip" data-s-tooltip-placement="right" data-s-popover-placement="" title="Save this question." data-is-saved="false" aria-label="Save" data-post-id="37503" data-post-type-id="1" data-user-privilege-for-post-click="0" aria-controls="" data-s-popover-auto-show="false" > <svg aria-hidden="true" class="fc-theme-primary-400 js-saves-btn-selected d-none svg-icon iconBookmark" width="18" height="18" viewBox="0 0 18 18"><path d="M3 17V3c0-1.1.9-2 2-2h8a2 2 0 0 1 2 2v14l-6-4z"/></svg> <svg aria-hidden="true" class="js-saves-btn-unselected svg-icon iconBookmarkAlt" width="18" height="18" viewBox="0 0 18 18"><path d="m9 10.6 4 2.66V3H5v10.26zM3 17V3c0-1.1.9-2 2-2h8a2 2 0 0 1 2 2v14l-6-4z"/></svg> </button> <a class="js-post-issue flex--item s-btn s-btn__unset c-pointer py6 mx-auto" href="/posts/37503/timeline" data-shortcut="T" data-ks-title="timeline" data-controller="s-tooltip" data-s-tooltip-placement="right" title="Show activity on this post." aria-label="Timeline"><svg aria-hidden="true" class="mln2 mr0 svg-icon iconHistory" width="19" height="18" viewBox="0 0 19 18"><path d="M3 9a8 8 0 1 1 3.73 6.77L8.2 14.3A6 6 0 1 0 5 9l3.01-.01-4 4-4-4zm7-4h1.01L11 9.36l3.22 2.1-.6.93L10 10z"/></svg></a> </div> </div> <div class="postcell post-layout--right"> <div class="s-prose js-post-body" itemprop="text"> <p>Having irregular verbs makes the language more complex. Users have to memorize more rules.</p> <p>Is there a historical reason, or some other reason, that English had all these irregular verbs?</p> </div> <div class="mt24 mb12"> <div class="post-taglist d-flex gs4 gsy fd-column"> <div class="d-flex ps-relative fw-wrap"> <ul class='ml0 list-ls-none js-post-tag-list-wrapper d-inline'><li class='d-inline mr4 js-post-tag-list-item'><a href="/questions/tagged/verbs" class="s-tag post-tag" title="show questions tagged 'verbs'" aria-label="show questions tagged 'verbs'" rel="tag" aria-labelledby="tag-verbs-tooltip-container" data-tag-menu-origin="Unknown">verbs</a></li><li class='d-inline mr4 js-post-tag-list-item'><a href="/questions/tagged/irregularity" class="s-tag post-tag" title="show questions tagged 'irregularity'" aria-label="show questions tagged 'irregularity'" rel="tag" aria-labelledby="tag-irregularity-tooltip-container" data-tag-menu-origin="Unknown">irregularity</a></li></ul> </div> </div> </div> <div class="mb0 "> <div class="mt16 d-flex gs8 gsy fw-wrap jc-end ai-start pt4 mb16"> <div class="flex--item mr16 fl1 w96"> <div class="js-post-menu pt2" data-post-id="37503" data-post-type-id="1"> <div class="d-flex gs8 s-anchors s-anchors__muted fw-wrap"> <div class="flex--item"> <a href="/q/37503" rel="nofollow" itemprop="url" class="js-share-link js-gps-track" title="Short permalink to this question" data-gps-track="post.click({ item: 2, priv: 0, post_type: 1 })" data-controller="se-share-sheet" data-se-share-sheet-title="Share a link to this question" data-se-share-sheet-subtitle="" data-se-share-sheet-post-type="question" data-se-share-sheet-social="facebook twitter " data-se-share-sheet-location="1" data-se-share-sheet-license-url="https%3a%2f%2fcreativecommons.org%2flicenses%2fby-sa%2f4.0%2f" data-se-share-sheet-license-name="CC BY-SA 4.0" data-s-popover-placement="bottom-start">Share</a> </div> <div class="flex--item"> <a href="/posts/37503/edit" class="js-suggest-edit-post js-gps-track" data-gps-track="post.click({ item: 6, priv: 0, post_type: 1 })" title="">Improve this question</a> </div> <div class="flex--item"> <button type="button" id="btnFollowPost-37503" class="s-btn s-btn__link js-follow-post js-follow-question js-gps-track" data-gps-track="post.click({ item: 14, priv: 0, post_type: 1 })" data-controller="s-tooltip " data-s-tooltip-placement="bottom" data-s-popover-placement="bottom" aria-controls="" title="Follow this question to receive notifications"> Follow <input type="hidden" id="voteFollowHash" value="67:3:31e,16:7c45694a23e37401,10:1739733043,16:090dd57dc07bfe02,5:37503,91be1c33672c25c64a3f7dde736b7524154e7c2428af4cab94eedcddebcc17bb" /> </button> </div> </div> <div class="js-menu-popup-container"></div> </div> </div> <div class="post-signature owner flex--item"> <div class="user-info "> <div class="d-flex "> <div class="user-action-time fl-grow1"> asked <span title='2020-10-31 03:31:23Z' class='relativetime'>Oct 31, 2020 at 3:31</span> </div> </div> <div class="user-gravatar32"> <a href="/users/30923/j-li"><div class="gravatar-wrapper-32"><img src="https://www.gravatar.com/avatar/ebb25608ed519e02eb7a4fe8dc5cd3dd?s=64&d=identicon&r=PG" alt="J Li's user avatar" width="32" height="32" class="bar-sm"></div></a> </div> <div class="user-details" itemprop="author" itemscope itemtype="http://schema.org/Person"> <a href="/users/30923/j-li" dir="auto">J Li</a><span class="d-none" itemprop="name">J Li</span> <div class="-flair"> <span class="reputation-score" title="reputation score " dir="ltr">597</span><span title="3 silver badges" aria-hidden="true"><span class="badge2"></span><span class="badgecount">3</span></span><span class="v-visible-sr">3 silver badges</span><span title="7 bronze badges" aria-hidden="true"><span class="badge3"></span><span class="badgecount">7</span></span><span class="v-visible-sr">7 bronze badges</span> </div> </div> </div> </div> </div> </div> </div> <span class="d-none" itemprop="commentCount">3</span> <div class="post-layout--right js-post-comments-component"> <div id="comments-37503" class="comments js-comments-container bt bc-black-200 mt12 " data-post-id="37503" data-min-length="15"> <ul class="comments-list js-comments-list" data-remaining-comments-count="0" data-canpost="false" data-cansee="true" data-comments-unavailable="false" data-addlink-disabled="true"> <li id="comment-85872" class="comment js-comment " data-comment-id="85872" data-comment-owner-id="2253" data-comment-score="2"> <div class="js-comment-actions comment-actions"> <div class="comment-score js-comment-score js-comment-edit-hide"> <span title="number of 'useful comment' votes received" class="cool">2</span> </div> </div> <div class="comment-text js-comment-text-and-form"> <div class="comment-body js-comment-edit-hide"> <span class="comment-copy">All natural languages that have verbal morphology also have verbs that can be termed ‘irregular’. Even many analytic languages like Mandarin have verbs that can be termed ‘irregular’ (e.g., non-past verbs are normally negated with 不 <i>bù</i>, except for the verb 有 <i>yǒu</i> ‘have’ which instead uses 没 <i>méi</i>). Some languages have more irregular verbs than others; that’s just a fact of languages. It’s usually a simple artefact of language change. It doesn’t really mean that ‘users’ (= native speakers) have to “memorize more rules”, though – they just speak. Only non-native speakers memorise rules.</span> <div class="d-inline-flex ai-center"> – <a href="/users/2253/janus-bahs-jacquet" title="5,476 reputation" class="comment-user">Janus Bahs Jacquet</a> </div> <span class="comment-date" dir="ltr"> <span class="v-visible-sr">Commented</span> <span title='2020-10-31 10:08:47Z, License: CC BY-SA 4.0' class='relativetime-clean'>Oct 31, 2020 at 10:08</span> </span> </div> </div> </li> <li id="comment-85874" class="comment js-comment " data-comment-id="85874" data-comment-owner-id="445" data-comment-score="5"> <div class="js-comment-actions comment-actions"> <div class="comment-score js-comment-score js-comment-edit-hide"> <span title="number of 'useful comment' votes received" class="warm">5</span> </div> </div> <div class="comment-text js-comment-text-and-form"> <div class="comment-body js-comment-edit-hide"> <span class="comment-copy">It’s explained here <a href="https://en.m.wikipedia.org/wiki/English_irregular_verbs#Development" rel="nofollow noreferrer">en.m.wikipedia.org/wiki/English_irregular_verbs#Development</a></span> <div class="d-inline-flex ai-center"> – <a href="/users/445/alex-b" title="9,046 reputation" class="comment-user">Alex B.</a> </div> <span class="comment-date" dir="ltr"> <span class="v-visible-sr">Commented</span> <span title='2020-10-31 12:49:34Z, License: CC BY-SA 4.0' class='relativetime-clean'>Oct 31, 2020 at 12:49</span> </span> </div> </div> </li> <li id="comment-86230" class="comment js-comment " data-comment-id="86230" data-comment-owner-id="30923" data-comment-score="0"> <div class="js-comment-actions comment-actions"> <div class="comment-score js-comment-score js-comment-edit-hide"> </div> </div> <div class="comment-text js-comment-text-and-form"> <div class="comment-body js-comment-edit-hide"> <span class="comment-copy">Janus and Alex - thank you for your answers. I guess my question can be better phrased to ask why did complex conjugation rules existed in the first place. After all, they are more complex and harder to use, and one would imagine that simpler rules would prevail if a language arose for communication purposes?</span> <div class="d-inline-flex ai-center"> – <a href="/users/30923/j-li" title="597 reputation" class="comment-user owner">J Li</a> </div> <span class="comment-date" dir="ltr"> <span class="v-visible-sr">Commented</span> <span title='2020-11-18 18:13:39Z, License: CC BY-SA 4.0' class='relativetime-clean'>Nov 18, 2020 at 18:13</span> </span> </div> </div> </li> </ul> </div> <div id="comments-link-37503" data-rep=50 data-anon=true> <a class="js-add-link comments-link disabled-link" title="Use comments to ask for more information or suggest improvements. Avoid answering questions in comments." href="#" role="button">Add a comment</a> <span class="js-link-separator dno"> | </span> <a class="js-show-link comments-link dno" title="Expand to show all comments on this post" href=# onclick="" role="button"></a> </div> </div> <script defer src="https://cdn.sstatic.net/Js/webpack-chunks/svelte.en.js?v=ac1681a633a2"></script><script defer src="https://cdn.sstatic.net/Js/webpack-chunks/stacks-svelte.en.js?v=585200d05e44"></script><script defer src="https://cdn.sstatic.net/Js/webpack-chunks/2784.en.js?v=414ac3934ec2"></script><script defer src="https://cdn.sstatic.net/Js/islands/comment-popover.en.js?v=96bfd850c9e6"></script> </div> </div> <div id="answers"> <a name="tab-top"></a> <div id="answers-header"> <div class="answers-subheader d-flex ai-center mb8"> <div class="flex--item fl1"> <h2 class="mb0" data-answercount="1"> 1 Answer <span style="display:none;" itemprop="answerCount">1</span> </h2> </div> <div class="flex--item"> <div class="d-flex g4 gsx ai-center sm:fd-column sm:ai-start"> <div class="d-flex fd-column ai-end sm:ai-start"> <label class="flex--item fs-caption" for="answer-sort-dropdown-select-menu"> Sorted by: </label> <a class="js-sort-preference-change s-link flex--item fs-fine d-none" data-value="ScoreDesc" href="/questions/37503/what-is-the-reason-for-having-irregular-verbs?answertab=scoredesc#tab-top" > Reset to default </a> </div> <div class="flex--item s-select"> <select id="answer-sort-dropdown-select-menu"> <option value=scoredesc selected=selected > Highest score (default) </option> <option value=modifieddesc > Date modified (newest first) </option> <option value=createdasc > Date created (oldest first) </option> </select> </div> </div> </div> </div> </div> <a name="37518"></a> <div id="answer-37518" class="answer js-answer" data-answerid="37518" data-parentid="37503" data-score="6" data-position-on-page="1" data-highest-scored="1" data-question-has-accepted-highest-score="0" itemprop="suggestedAnswer" itemscope itemtype="https://schema.org/Answer"> <div class="post-layout"> <div class="votecell post-layout--left"> <div class="js-voting-container d-flex jc-center fd-column ai-center gs4 fc-black-300" data-post-id="37518" data-referrer="None"> <button class="js-vote-up-btn flex--item s-btn s-btn__muted s-btn__outlined bar-pill bc-black-225 f:bc-theme-secondary-400 f:bg-theme-secondary-400 f:fc-black-050 h:bg-theme-primary-200" id="upvote-btn-37518" data-controller="s-tooltip" data-s-tooltip-placement="right" title="This answer is useful" aria-pressed="false" aria-label="Up vote" data-selected-classes="fc-theme-primary-100 bc-theme-primary-500 bg-theme-primary-500" data-unselected-classes="bc-black-225 f:bc-theme-secondary-400 f:bg-theme-secondary-400 f:fc-black-050 h:bg-theme-primary-200"> <svg aria-hidden="true" class="svg-icon iconArrowUp" width="18" height="18" viewBox="0 0 18 18"><path d="M1 12h16L9 4z"/></svg> </button> <input type="hidden" id="voteUpHash" value="67:3:31e,16:6048482bda2fa4bf,10:1739733043,16:634134b626640211,5:37518,ba38d9a7290968ebd4601c598581696f884e8ae650132a3fd4022918ce5bb3db" /> <div class="js-vote-count flex--item d-flex fd-column ai-center fc-theme-body-font fw-bold fs-subheading py4" itemprop="upvoteCount" data-value="6"> 6 </div> <button class="js-vote-down-btn flex--item mb8 s-btn s-btn__muted s-btn__outlined bar-pill bc-black-225 f:bc-theme-secondary-400 f:bg-theme-secondary-400 f:fc-black-050 h:bg-theme-primary-200" title="This answer is not useful" aria-pressed="false" aria-label="Down vote" data-selected-classes="fc-theme-primary-100 bc-theme-primary-500 bg-theme-primary-500" data-unselected-classes="bc-black-225 f:bc-theme-secondary-400 f:bg-theme-secondary-400 f:fc-black-050 h:bg-theme-primary-200"> <svg aria-hidden="true" class="svg-icon iconArrowDown" width="18" height="18" viewBox="0 0 18 18"><path d="M1 6h16l-8 8z"/></svg> </button> <input type="hidden" id="voteDownHash" value="67:3:31e,16:277ef89265dc6a30,10:1739733043,16:dfb7edbf7c9e77ec,5:37518,3b0c1f3bfcf2af92cfb2930e26977bfa422aae2f6f51acb72d7bb741e5041f76" /> <button class="js-saves-btn s-btn s-btn__unset c-pointer py4" type="button" id="saves-btn-37518" data-controller="s-tooltip" data-s-tooltip-placement="right" data-s-popover-placement="" title="Save this answer." data-is-saved="false" aria-label="Save" data-post-id="37518" data-post-type-id="2" data-user-privilege-for-post-click="0" aria-controls="" data-s-popover-auto-show="false" > <svg aria-hidden="true" class="fc-theme-primary-400 js-saves-btn-selected d-none svg-icon iconBookmark" width="18" height="18" viewBox="0 0 18 18"><path d="M3 17V3c0-1.1.9-2 2-2h8a2 2 0 0 1 2 2v14l-6-4z"/></svg> <svg aria-hidden="true" class="js-saves-btn-unselected svg-icon iconBookmarkAlt" width="18" height="18" viewBox="0 0 18 18"><path d="m9 10.6 4 2.66V3H5v10.26zM3 17V3c0-1.1.9-2 2-2h8a2 2 0 0 1 2 2v14l-6-4z"/></svg> </button> <div class="js-accepted-answer-indicator flex--item fc-green-400 py6 mtn8 d-none" data-s-tooltip-placement="right" title="Loading when this answer was accepted…" tabindex="0" role="note" aria-label="Accepted"> <div class="ta-center"> <svg aria-hidden="true" class="svg-icon iconCheckmarkLg" width="36" height="36" viewBox="0 0 36 36"><path d="m6 14 8 8L30 6v8L14 30l-8-8z"/></svg> </div> </div> <a class="js-post-issue flex--item s-btn s-btn__unset c-pointer py6 mx-auto" href="/posts/37518/timeline" data-shortcut="T" data-ks-title="timeline" data-controller="s-tooltip" data-s-tooltip-placement="right" title="Show activity on this post." aria-label="Timeline"><svg aria-hidden="true" class="mln2 mr0 svg-icon iconHistory" width="19" height="18" viewBox="0 0 19 18"><path d="M3 9a8 8 0 1 1 3.73 6.77L8.2 14.3A6 6 0 1 0 5 9l3.01-.01-4 4-4-4zm7-4h1.01L11 9.36l3.22 2.1-.6.93L10 10z"/></svg></a> </div> </div> <div class="answercell post-layout--right"> <div class="s-prose js-post-body" itemprop="text"> <p>Just as in biological evolution you end up with vestigial limbs, or organs that no longer serve any purpose, or body plans that seem wildly impractical (e.g. the recurrent laryngeal nerve, connecting the brain to the larynx but looping around the heart, or the fact the retina's blood supply is in front of it, not behind), so too do languages have irregularities. It's just the detritus that comes from having evolved naturally over time</p> <p>There are three main sources of irregularities, and from a diachronic (i.e. looking at the entire history of the language), only one really looks properly irregular. The other two are just unusual. Of course, many words may have aspects of more than one of these going on</p> <ol> <li>Sound changes have unexpected side effects: <ul> <li>As a language evolves its sounds do too. These changes can be either conditioned (where the outcome of a segment depends on the surrounding segments), or unconditioned. Sometimes these changes lead to alternations in a paradigm as different sound changes apply to different parts</li> <li>e.g. in the history of Spanish, Latin short e stays as e when unstressed (merging with Latin short i, and long ē), but becomes ie when stressed. Because Latin stress depends on the structure of the final two syllables (the rules used to be more complicated, but at the point this change applies, stress was fixed on the penultimate syllable), and verb endings have quite a varied number and shape, the stress moves around between the ending and the stem. This is the cause of stem-changing verbs e.g. contar "to count" - cuento "I count" - contamos "we count" < Latin computāre - computō - computāmus (via a regularly expected intermediate *còntarè - cònto - còntamos, with the grave accents representing the lower o & e descended from the short vowel)</li> </ul> </li> <li>Retention of old paradigms: <ul> <li>Sometimes stops using a certain morphological paradigm. Maybe it stops using the words that belong to it, or maybe words belonging to that paradigm get reanalysed as belonging to a different paradigm and adapted to fit it. Regardless, a few words are likely to be left behind inflecting in a way that now seems irregular</li> <li>e.g. Like German, Middle English retained a rudimentary case system, as well as a distinction between "strong" and "weak" nouns. Strong nouns form a genitive singular, and nominative/accusative plural with -es (this is the paradigm that came to dominate in Modern English, although we now distinguish between these two -es'es in writing), whilst weak nouns form all plurals and genitives with the the suffix -en. This survives in a few irregular plurals, notably ox - oxen</li> <li>Combined with type 1, this gives us things like English strong verbs (most verbs that people mean when they say "irregular" verbs in English). Proto-Germanic had already innovated a new way of forming a past tense, with a suffix in -d- (which gives Modern English -ed), but some verbs retain the earlier Proto-Indo-European system of ablaut. In this system, the vowel in the stem changes to form the past tense (and a slightly different set of endings are used). In Proto-Germanic, sound changes had already made the Proto-Indo-European ablaut patterns rather complicated (there are 7 classes of strong verb, some of which have subclasses), but they can still be traced moreorless directly back to a simpler PIE pattern. Unfortunately, by Modern English sound changes have made the patterns almost indiscernible and reanalysis has moved verbs between classes, made them regular or, in a few cases, even innovated new patterns of ablaut entirely</li> <li>English umlaut plurals (e.g. goose - geese) also come from a mix of type 1 & 2. In Proto-West-Germanic these were a form of neuter that formed its nominative plural by adding an -i ending. In a process called I-umlaut, this ending caused the preceding vowel to move forwards in the mouth and raise. This -i was later lost, but the umlaut remained. Despite this paradigm having generally fallen out of use (replaced with the usual -s plural), some words retain it</li> </ul> </li> <li>Suppletion: <ul> <li>This is the most truly irregular type, and is also the rarest. It's not just the sound of words that changes, but also their meaning. Through a process called semantic bleaching, an originally precise term can acquire a very broad and general one. This often leads to several words existing without a clearcut semantic distinction between them. Language abhors a true equivalents though so inevitably a pattern emerges with people using one form more often in some environments than the other. If the distinction is semantic or pragmatic, the meanings will just diverge, but often the distinction is morphosyntactic. Once this pattern has emerged and been noticed (even subconsciously) it becomes self-reinforcing as it becomes more and more marked to go against it. Eventually, this distinction becomes grammaticalised, with one synonym used in part of the paradigm, and another synonym used in another</li> <li>e.g. the verb "to be" has undergone this process twice. There are three sources of form in this verb, the b-forms (e.g. "be", "being", "been", used in non-finite, and imperative forms only), the w-forms (e.g. "was", "were", used in the past), and the s-forms (e.g. "am", "is", "are", used in the present). These all come from different PIE verbs. The s-forms go back all the way to the original PIE verb for "to be" and cognates can be found in most Indo-European languages. They are highly irregular, retaining many otherwise lost aspects of the PIE verb paradigm, and with odd sound change side effects. By Old English though, the w-forms had already merged into this forming a single verb "wesan" which used s-forms in the present indicative and subjunctive and w-forms elsewhere. At this point, the b-forms were still a separate verb "bēon" (the distinction was similar to, although not quite the same as, that between Spanish "ser" and "estar", with "bēon" being more like "ser") which later merged in during Middle English</li> <li>English has only a handful suppletive verbs (not counting verbs transparently derived from them) and possibly only a single suppletive noun: person - people (although in certain circumstances, plural persons, singular people, and plural peoples are all acceptable)</li> </ul> </li> </ol> <p>So irregularities get built up, but why not get rid of them? Why bother with all this nonsense instead of just saying "the persons' oxes be next to the gooses"?</p> <p>Because it sounds weird</p> <p>Making such a dramatic alteration to your speech requires conscious thought, and makes you stand out, and most people don't really want either when they're just going about their daily lives. And so if they're always hearing an irregular form be used, they're probably going to keep using it</p> <p>Of course, if they don't hear the irregular form very often, they're less likely to remember the proper irregular form, and may default to a regular one. Over time, this form may even win out and the irregular form will have been regularised. This is why common verbs (especially "to be") are more likely to be irregular. It's not that frequent use makes them more likely to develop irregularities, just that it makes them more likely to retain those that they do develop</p> </div> <div class="mt24"> <div class="d-flex fw-wrap ai-start jc-end gs8 gsy"> <time itemprop="dateCreated" datetime="2020-11-02T12:15:50"></time> <div class="flex--item mr16" style="flex: 1 1 100px;"> <div class="js-post-menu pt2" data-post-id="37518" data-post-type-id="2"> <div class="d-flex gs8 s-anchors s-anchors__muted fw-wrap"> <div class="flex--item"> <a href="/a/37518" rel="nofollow" itemprop="url" class="js-share-link js-gps-track" title="Short permalink to this answer" data-gps-track="post.click({ item: 2, priv: 0, post_type: 2 })" data-controller="se-share-sheet" data-se-share-sheet-title="Share a link to this answer" data-se-share-sheet-subtitle="" data-se-share-sheet-post-type="answer" data-se-share-sheet-social="facebook twitter " data-se-share-sheet-location="2" data-se-share-sheet-license-url="https%3a%2f%2fcreativecommons.org%2flicenses%2fby-sa%2f4.0%2f" data-se-share-sheet-license-name="CC BY-SA 4.0" data-s-popover-placement="bottom-start">Share</a> </div> <div class="flex--item"> <a href="/posts/37518/edit" class="js-suggest-edit-post js-gps-track" data-gps-track="post.click({ item: 6, priv: 0, post_type: 2 })" title="">Improve this answer</a> </div> <div class="flex--item"> <button type="button" id="btnFollowPost-37518" class="s-btn s-btn__link js-follow-post js-follow-answer js-gps-track" data-gps-track="post.click({ item: 14, priv: 0, post_type: 2 })" data-controller="s-tooltip " data-s-tooltip-placement="bottom" data-s-popover-placement="bottom" aria-controls="" title="Follow this answer to receive notifications"> Follow <input type="hidden" id="voteFollowHash" value="67:3:31e,16:3ebaeda7edf1f098,10:1739733043,16:08e9ec9c07d9c2fd,5:37518,ce4d40688878d24bb8571417b0b715dabeabf73f911f94a28fb35b29762eb1f5" /> </button> </div> </div> <div class="js-menu-popup-container"></div> </div> </div> <div class="post-signature flex--item fl0"> <div class="user-info "> <div class="d-flex "> <div class="user-action-time fl-grow1"> answered <span title='2020-11-02 12:15:50Z' class='relativetime'>Nov 2, 2020 at 12:15</span> </div> </div> <div class="user-gravatar32"> <a href="/users/28236/tristan"><div class="gravatar-wrapper-32"><img src="https://www.gravatar.com/avatar/a0a7d2a892d906f9098b1e8211b2687c?s=64&d=identicon&r=PG&f=y&so-version=2" alt="Tristan's user avatar" width="32" height="32" class="bar-sm"></div></a> </div> <div class="user-details" itemprop="author" itemscope itemtype="http://schema.org/Person"> <a href="/users/28236/tristan" dir="auto">Tristan</a><span class="d-none" itemprop="name">Tristan</span> <div class="-flair"> <span class="reputation-score" title="reputation score 10,434" dir="ltr">10.4k</span><span title="26 silver badges" aria-hidden="true"><span class="badge2"></span><span class="badgecount">26</span></span><span class="v-visible-sr">26 silver badges</span><span title="53 bronze badges" aria-hidden="true"><span class="badge3"></span><span class="badgecount">53</span></span><span class="v-visible-sr">53 bronze badges</span> </div> </div> </div> </div> </div> </div> </div> <span class="d-none" itemprop="commentCount">3</span> <div class="post-layout--right js-post-comments-component"> <div id="comments-37518" class="comments js-comments-container bt bc-black-200 mt12 " data-post-id="37518" data-min-length="15"> <ul class="comments-list js-comments-list" data-remaining-comments-count="0" data-canpost="false" data-cansee="true" data-comments-unavailable="false" data-addlink-disabled="true"> <li id="comment-85949" class="comment js-comment " data-comment-id="85949" data-comment-owner-id="482" data-comment-score="1"> <div class="js-comment-actions comment-actions"> <div class="comment-score js-comment-score js-comment-edit-hide"> <span title="number of 'useful comment' votes received" class="cool">1</span> </div> </div> <div class="comment-text js-comment-text-and-form"> <div class="comment-body js-comment-edit-hide"> <span class="comment-copy">Somebody's paradox (Sapir?) is that sound change is regular but causes irregularity in paradigms, while analogic change is irregular but produces regularity in paradigms.</span> <div class="d-inline-flex ai-center"> – <a href="/users/482/jlawler" title="10,132 reputation" class="comment-user">jlawler</a> </div> <span class="comment-date" dir="ltr"> <span class="v-visible-sr">Commented</span> <span title='2020-11-02 19:46:17Z, License: CC BY-SA 4.0' class='relativetime-clean'>Nov 2, 2020 at 19:46</span> </span> </div> </div> </li> <li id="comment-85962" class="comment js-comment " data-comment-id="85962" data-comment-owner-id="22504" data-comment-score="1"> <div class="js-comment-actions comment-actions"> <div class="comment-score js-comment-score js-comment-edit-hide"> <span title="number of 'useful comment' votes received" class="cool">1</span> </div> </div> <div class="comment-text js-comment-text-and-form"> <div class="comment-body js-comment-edit-hide"> <span class="comment-copy">Of course you need to ask now, 1. what's the reason behind sound change, whether it's a different reason the same one. The latter would imply that there's no reason for irregular verbs, they come about arbitrary. 2. was PIE Ablaut or the many classes of verbs any less arbitrary, 3. is this a good explanation of suppletive paradimgs assuming they are a combination of various fully fledged paradigms, or did some of these verbs only form in relevant instances and retain the meaning throughout (I am, of course, referring to preverbs and auxiliaries, first of all).</span> <div class="d-inline-flex ai-center"> – <a href="/users/22504/vectory" title="1,383 reputation" class="comment-user">vectory</a> </div> <span class="comment-date" dir="ltr"> <span class="v-visible-sr">Commented</span> <span title='2020-11-03 16:17:20Z, License: CC BY-SA 4.0' class='relativetime-clean'>Nov 3, 2020 at 16:17</span> </span> </div> </div> </li> <li id="comment-85963" class="comment js-comment " data-comment-id="85963" data-comment-owner-id="22504" data-comment-score="0"> <div class="js-comment-actions comment-actions"> <div class="comment-score js-comment-score js-comment-edit-hide"> </div> </div> <div class="comment-text js-comment-text-and-form"> <div class="comment-body js-comment-edit-hide"> <span class="comment-copy">Despite this, it exposes the idea of change nicely. Yet, conclusing that change doesn't tend towards regularity, because it would be too much work, is not convincing. It's ironically correct in a sense, because a true answer would take too much work. First, the greatest opportunity would be in language acquisition -- even in 2nd acquisition, I don't <i>see</i> the problem with <i>oxes</i> (with stress on <i>see</i>), nor <i>octopusses</i> (standard, over Greek). Second, the problem must then be in reception and acceptance. So, old farts are literally incapable of change, and if they aren't, they are too lazy? No.</span> <div class="d-inline-flex ai-center"> – <a href="/users/22504/vectory" title="1,383 reputation" class="comment-user">vectory</a> </div> <span class="comment-date" dir="ltr"> <span class="v-visible-sr">Commented</span> <span title='2020-11-03 16:26:50Z, License: CC BY-SA 4.0' class='relativetime-clean'>Nov 3, 2020 at 16:26</span> </span> <span title="this comment was edited 1 time"> <svg aria-hidden="true" class="va-text-bottom o50 svg-icon iconPencilSm" width="14" height="14" viewBox="0 0 14 14"><path fill="#F1B600" d="m2 10.12 6.37-6.43 1.88 1.88L3.88 12H2z"/><path fill="#E87C87" d="m11.1 1.71 1.13 1.12c.2.2.2.51 0 .71L11.1 4.7 9.21 2.86l1.17-1.15c.2-.2.51-.2.71 0"/></svg> </span> </div> </div> </li> </ul> </div> <div id="comments-link-37518" data-rep=50 data-anon=true> <a class="js-add-link comments-link disabled-link" title="Use comments to ask for more information or suggest improvements. Avoid comments like “+1” or “thanks”." href="#" role="button">Add a comment</a> <span class="js-link-separator dno"> | </span> <a class="js-show-link comments-link dno" title="Expand to show all comments on this post" href=# onclick="" role="button"></a> </div> </div> </div> </div> <a name='new-answer'></a> <form id="post-form" action="/questions/37503/answer/submit" method="post" class="js-add-answer-component post-form"> <input type="hidden" id="post-id" value="37503" /> <input type="hidden" id="qualityBanWarningShown" name="qualityBanWarningShown" value="false" /> <input type="hidden" name="referrer" value="" /> <h2 class="space" id="your-answer-header"> Your Answer </h2> <script> StackExchange.ready(function() { var channelOptions = { tags: "".split(" "), id: "312" }; initTagRenderer("".split(" "), "".split(" "), channelOptions); StackExchange.using("externalEditor", function() { // Have to fire editor after snippets, if snippets enabled if (StackExchange.settings.snippets.snippetsEnabled) { StackExchange.using("snippets", function() { createEditor(); }); } else { createEditor(); } }); function createEditor() { StackExchange.prepareEditor({ useStacksEditor: false, heartbeatType: 'answer', autoActivateHeartbeat: false, convertImagesToLinks: false, noModals: true, showLowRepImageUploadWarning: true, reputationToPostImages: null, bindNavPrevention: true, postfix: "", imageUploadEnabled: false, imageUploader: { brandingHtml: "", contentPolicyHtml: "User contributions licensed under \u003ca href=\"https://stackoverflow.com/help/licensing\"\u003eCC BY-SA\u003c/a\u003e \u003ca href=\"https://stackoverflow.com/legal/acceptable-use-policy\"\u003e(content policy)\u003c/a\u003e", allowUrls: true, }, noCode: true, onDemand: true, discardSelector: ".discard-answer", enableTables: true, isStacksEditorPreviewEnabled: false ,immediatelyShowMarkdownHelp:true,enableTables:true }); } }); </script> <div id="post-editor" class="post-editor js-post-editor d-flex fd-column g4"> <input type="hidden" name="stacksEditorEnabled" value="False" /> <div class="ps-relative"> <div class="wmd-container mb8"> <div id="wmd-button-bar" class="wmd-button-bar btr-sm"></div> <div class="js-stacks-validation"> <div class="ps-relative"> <textarea id="wmd-input" name="post-text" class="wmd-input s-input bar0 js-post-body-field" data-editor-type="wmd" data-post-type-id="2" cols="92" rows="15" aria-labelledby="your-answer-header" tabindex="101" data-min-length=""></textarea> </div> <div class="s-input-message mt4 d-none js-stacks-validation-message"></div> </div> </div> </div> <aside class="d-flex ai-start jc-space-between js-answer-help s-notice s-notice__warning pb0 pr4 pt4 mb8 d-none" role="status" aria-hidden="true"> <div class="flex--item pt8"> <p>Thanks for contributing an answer to Linguistics Stack Exchange!</p><ul><li>Please be sure to <em>answer the question</em>. Provide details and share your research!</li></ul><p>But <em>avoid</em> …</p><ul><li>Asking for help, clarification, or responding to other answers.</li><li>Making statements based on opinion; back them up with references or personal experience.</li></ul><p>To learn more, see our <a href="/help/how-to-answer">tips on writing great answers</a>.</p> </div> <button class="flex--item js-answer-help-close-btn s-btn s-btn__muted fc-black-600"> <svg aria-hidden="true" class="svg-icon iconClear" width="18" height="18" viewBox="0 0 18 18"><path d="M15 4.41 13.59 3 9 7.59 4.41 3 3 4.41 7.59 9 3 13.59 4.41 15 9 10.41 13.59 15 15 13.59 10.41 9z"/></svg> </button> </aside> <div> <div id="draft-saved" class="fc-success h24" style="display:none;">Draft saved</div> <div id="draft-discarded" class="fc-error h24" style="display:none;">Draft discarded</div> </div> <div id="wmd-preview" class="s-prose mb16 wmd-preview js-wmd-preview"></div> <div></div> <div class="edit-block"> <input id="fkey" name="fkey" type="hidden" value="b39cb1a761f93912de068f8e6267ee5702b7d597a570755c5112354b099addd4"> <input id="author" name="author" type="text"> </div> </div> <div class="ps-relative"> <div class="form-item dno new-post-login p0 my16"> <div class="d-flex gs16 md:fd-column new-login-form"> <div class="d-flex fd-column w50 md:w-auto gsy gs8 jc-space-between new-login-left"> <h3 class="flex--item fs-title">Sign up or <a id="login-link" href="/users/login?ssrc=question_page&returnurl=https%3a%2f%2flinguistics.stackexchange.com%2fquestions%2f37503%2fwhat-is-the-reason-for-having-irregular-verbs%23new-answer">log in</a></h3> <script> StackExchange.ready(function () { StackExchange.helpers.onClickDraftSave('#login-link'); }); </script> <div class="flex--item s-btn s-btn__muted s-btn__outlined s-btn__icon google-login" data-ga="["sign up","Sign Up Started - Google","New Post",null,null]"> <svg aria-hidden="true" class="native svg-icon iconGoogle" width="18" height="18" viewBox="0 0 18 18"><path fill="#4285F4" d="M16.51 8H8.98v3h4.3c-.18 1-.74 1.48-1.6 2.04v2.01h2.6a7.8 7.8 0 0 0 2.38-5.88c0-.57-.05-.66-.15-1.18"/><path fill="#34A853" d="M8.98 17c2.16 0 3.97-.72 5.3-1.94l-2.6-2a4.8 4.8 0 0 1-7.18-2.54H1.83v2.07A8 8 0 0 0 8.98 17"/><path fill="#FBBC05" d="M4.5 10.52a4.8 4.8 0 0 1 0-3.04V5.41H1.83a8 8 0 0 0 0 7.18z"/><path fill="#EA4335" d="M8.98 4.18c1.17 0 2.23.4 3.06 1.2l2.3-2.3A8 8 0 0 0 1.83 5.4L4.5 7.49a4.8 4.8 0 0 1 4.48-3.3"/></svg> Sign up using Google </div> <div class="flex--item s-btn s-btn__muted s-btn__outlined s-btn__icon stackexchange-login" data-ga="["sign up","Sign Up Navigation","New Post",null,null]"> <svg aria-hidden="true" class="native svg-icon iconGlyphXSm" width="18" height="18" viewBox="0 0 18 18"><path fill="#BCBBBB" d="M14 16v-5h2v7H2v-7h2v5z"/><path fill="#F48024" d="m12.09.72-1.21.9 4.5 6.07 1.22-.9zM5 15h8v-2H5zm9.15-5.87L8.35 4.3l.96-1.16 5.8 4.83zm-7.7-1.47 6.85 3.19.63-1.37-6.85-3.2zm6.53 5L5.4 11.39l.38-1.67 7.42 1.48z"/></svg> Sign up using Email and Password </div> </div> <input type="hidden" name="use-facebook" class="use-facebook" value="false" /> <input type="hidden" name="use-google" class="use-google" value="false" /> <button type="button" class="d-none js-submit-openid">Submit</button> <div class="d-flex gsy gs8 fd-column w50 md:w-auto new-login-right form-item p0"> <h3 class="flex--item fs-title">Post as a guest</h3> <div class="flex--item"> <div class="d-flex gs4 gsy fd-column"> <label class="s-label" for="display-name">Name</label> <div class="d-flex ps-relative"> <input class="s-input" id="display-name" name="display-name" maxlength="30" type="text" value="" tabindex="105" placeholder="" /> </div> </div> </div> <div class="flex--item"> <div class="d-flex gs4 gsy fd-column"> <div class="flex--item"> <div class="d-flex gs2 gsy fd-column"> <label class="flex--item s-label" for="m-address">Email</label> <p class="flex--item s-description">Required, but never shown</p> </div> </div> <div class="d-flex ps-relative"> <input class="s-input js-post-email-field" id="m-address" name="m-address" type="text" value="" size="40" tabindex="106" placeholder="" /> </div> </div> </div> </div> </div> </div> <script> StackExchange.ready( function () { StackExchange.openid.initPostLogin('.new-post-login', 'https%3a%2f%2flinguistics.stackexchange.com%2fquestions%2f37503%2fwhat-is-the-reason-for-having-irregular-verbs%23new-answer', 'question_page'); } ); </script> <noscript> <h3 class="flex--item fs-title">Post as a guest</h3> <div class="flex--item"> <div class="d-flex gs4 gsy fd-column"> <label class="s-label" for="display-name">Name</label> <div class="d-flex ps-relative"> <input class="s-input" id="display-name" name="display-name" maxlength="30" type="text" value="" tabindex="105" placeholder="" /> </div> </div> </div> <div class="flex--item"> <div class="d-flex gs4 gsy fd-column"> <div class="flex--item"> <div class="d-flex gs2 gsy fd-column"> <label class="flex--item s-label" for="m-address">Email</label> <p class="flex--item s-description">Required, but never shown</p> </div> </div> <div class="d-flex ps-relative"> <input class="s-input js-post-email-field" id="m-address" name="m-address" type="text" value="" size="40" tabindex="106" placeholder="" /> </div> </div> </div> </noscript> </div> <div class="form-submit clear-both d-flex sm:fd-column sm:jc-stretch gs4 ai-center"> <button id="submit-button" class="flex--item fl-shrink0 s-btn s-btn__filled sm:w100" type="submit" tabindex="120" autocomplete="off"> Post Your Answer </button> <button class="flex--item s-btn s-btn__danger fl-shrink0 sm:w100 discard-answer d-none"> Discard </button> <p class="flex--item mb0 fs-italic ml12 sm:ml0"> By clicking “Post Your Answer”, you agree to our <a href='https://stackoverflow.com/legal/terms-of-service/public' name='tos' target='_blank' class='-link'>terms of service</a> and acknowledge you have read our <a href='https://stackoverflow.com/legal/privacy-policy' name='privacy' target='_blank' class='-link'>privacy policy</a>.<input type="hidden" name="legalLinksShown" value="1" /> </p> </div> <div class="js-general-error general-error clear-both d-none" aria-live="polite"></div> </form> <h2 class="bottom-notice" data-loc="1"> <div> Not the answer you're looking for? Browse other questions tagged <ul class='ml0 list-ls-none js-post-tag-list-wrapper d-inline'><li class='d-inline mr4 js-post-tag-list-item'><a href="/questions/tagged/verbs" class="s-tag post-tag" title="show questions tagged 'verbs'" aria-label="show questions tagged 'verbs'" rel="tag" aria-labelledby="tag-verbs-tooltip-container" data-tag-menu-origin="Unknown">verbs</a></li><li class='d-inline mr4 js-post-tag-list-item'><a href="/questions/tagged/irregularity" class="s-tag post-tag" title="show questions tagged 'irregularity'" aria-label="show questions tagged 'irregularity'" rel="tag" aria-labelledby="tag-irregularity-tooltip-container" data-tag-menu-origin="Unknown">irregularity</a></li></ul> or <a href="/questions/ask">ask your own question</a>. </div> </h2> </div> </div> <div id="sidebar" class="show-votes" role="complementary" aria-label="sidebar"> <div class="s-sidebarwidget s-sidebarwidget__yellow s-anchors s-anchors__grayscale mb16" data-tracker="cb=1"> <ul class="s-sidebarwidget--content s-sidebarwidget__items p0"> <li class="s-sidebarwidget--header"> Featured on Meta </li> <li class="s-sidebarwidget--item d-flex px16"> <div class="flex--item1 fl-shrink0"> <div class="favicon favicon-stackexchangemeta" title="Meta Stack Exchange"></div> </div> <div class="flex--item wmn0 ow-break-word"> <a href="https://meta.stackexchange.com/questions/406575/bigbird-and-frog-have-joined-us-as-community-managers" class="js-gps-track" data-ga="["community bulletin board","Featured on Meta","https://meta.stackexchange.com/questions/406575/bigbird-and-frog-have-joined-us-as-community-managers",null,null]" data-gps-track="communitybulletin.click({ priority: 3, position: 0, location: questionpage })">bigbird and Frog have joined us as Community Managers</a> </div> </li> <li class="s-sidebarwidget--item d-flex px16"> <div class="flex--item1 fl-shrink0"> <div class="favicon favicon-stackexchangemeta" title="Meta Stack Exchange"></div> </div> <div class="flex--item wmn0 ow-break-word"> <a href="https://meta.stackexchange.com/questions/406399/join-us-for-our-first-community-wide-ama-ask-me-anything-with-stack-overflow-s" class="js-gps-track" title="Join us for our first community-wide AMA (Ask Me Anything) with Stack Overflow’s CEO (and a few others) on February 26, 2025" data-ga="["community bulletin board","Featured on Meta","https://meta.stackexchange.com/questions/406399/join-us-for-our-first-community-wide-ama-ask-me-anything-with-stack-overflow-s",null,null]" data-gps-track="communitybulletin.click({ priority: 3, position: 1, location: questionpage })">Join us for our first community-wide AMA (Ask Me Anything) with Stack...</a> </div> </li> <li class="s-sidebarwidget--item d-flex px16"> <div class="flex--item1 fl-shrink0"> <div class="favicon favicon-stackexchangemeta" title="Meta Stack Exchange"></div> </div> <div class="flex--item wmn0 ow-break-word"> <a href="https://meta.stackexchange.com/questions/406307/ai-generated-answers-experiment-on-stack-exchange-sites-that-volunteered-to-part" class="js-gps-track" title="AI-generated Answers experiment on Stack Exchange sites that volunteered to participate" data-ga="["community bulletin board","Featured on Meta","https://meta.stackexchange.com/questions/406307/ai-generated-answers-experiment-on-stack-exchange-sites-that-volunteered-to-part",null,null]" data-gps-track="communitybulletin.click({ priority: 3, position: 2, location: questionpage })">AI-generated Answers experiment on Stack Exchange sites that volunteered to...</a> </div> </li> </ul> </div> <div class="js-zone-container zone-container-sidebar"> <div id="dfp-tsb" class="everyonelovesstackoverflow everyoneloves__top-sidebar"></div> <div class="js-report-ad-button-container " style="width: 300px"></div> </div> <div class="module sidebar-related"> <h4 id="h-related">Related</h4> <div class="related js-gps-related-questions" data-tracker="rq=1"> <div class="spacer" data-question-id="2319"> <a href="/q/2319" title="Question score (upvotes - downvotes)" > <div class="answer-votes answered-accepted default">5</div> </a> <a href="/questions/2319/what-is-the-maximum-number-of-forms-a-modern-japanese-verb-can-take" class="question-hyperlink">What is the maximum number of forms a (modern) Japanese verb can take?</a> </div> <div class="spacer" data-question-id="6932"> <a href="/q/6932" title="Question score (upvotes - downvotes)" > <div class="answer-votes answered-accepted default">3</div> </a> <a href="/questions/6932/why-are-irregular-verbs-usually-common-words" class="question-hyperlink">Why are irregular verbs usually common words?</a> </div> <div class="spacer" data-question-id="9847"> <a href="/q/9847" title="Question score (upvotes - downvotes)" > <div class="answer-votes default">0</div> </a> <a href="/questions/9847/common-change-of-conjugation-of-the-verbs-in-spoken-languages" class="question-hyperlink">Common change of conjugation of the verbs in spoken languages?</a> </div> <div class="spacer" data-question-id="15488"> <a href="/q/15488" title="Question score (upvotes - downvotes)" > <div class="answer-votes default">0</div> </a> <a href="/questions/15488/did-all-regular-irregular-verbs-arise-from-the-same-two-sources" class="question-hyperlink">Did all regular/irregular verbs arise from the same two sources?</a> </div> <div class="spacer" data-question-id="29110"> <a href="/q/29110" title="Question score (upvotes - downvotes)" > <div class="answer-votes answered-accepted default">0</div> </a> <a href="/questions/29110/the-most-different-meanings-a-verb-has-been-found-to-have" class="question-hyperlink">The most different meanings a verb has been found to have</a> </div> <div class="spacer" data-question-id="31719"> <a href="/q/31719" title="Question score (upvotes - downvotes)" > <div class="answer-votes answered-accepted default">3</div> </a> <a href="/questions/31719/what-kind-of-verbs-take-three-arguments" class="question-hyperlink">What kind of verbs take three arguments?</a> </div> <div class="spacer" data-question-id="47990"> <a href="/q/47990" title="Question score (upvotes - downvotes)" > <div class="answer-votes answered-accepted default">3</div> </a> <a href="/questions/47990/why-is-the-future-tense-almost-completely-regular-in-portuguese" class="question-hyperlink">Why is the future tense almost completely regular in Portuguese?</a> </div> <div class="spacer" data-question-id="48113"> <a href="/q/48113" title="Question score (upvotes - downvotes)" > <div class="answer-votes default">2</div> </a> <a href="/questions/48113/is-there-a-reason-why-certain-verbs-use-certain-cases" class="question-hyperlink">Is there a reason why certain verbs use certain cases?</a> </div> </div> </div> <script type="text/javascript"> $(function() { $(".js-gps-related-questions .spacer").on("click", function () { fireRelatedEvent($(this).index() + 1, $(this).data('question-id')); }); function fireRelatedEvent(position, questionId) { StackExchange.using("gps", function() { StackExchange.gps.track('related_questions.click', { position: position, originQuestionId: 37503, relatedQuestionId: +questionId, location: 'sidebar', source: 'Baseline' }); }); } }); </script> <div id="hot-network-questions" class="module tex2jax_ignore"> <h4> <a href="https://stackexchange.com/questions?tab=hot" class="js-gps-track s-link s-link__inherit" data-gps-track="posts_hot_network.click({ item_type:1, location:11 })"> Hot Network Questions </a> </h4> <ul> <li > <div class="favicon favicon-law" title="Law Stack Exchange"></div><a href="https://law.stackexchange.com/questions/107407/is-a-person-free-to-say-anything-before-signing-an-nda" class="js-gps-track question-hyperlink mb0" data-gps-track="site.switch({ item_type:11, target_site:617 }); posts_hot_network.click({ item_type:2, location:11 })"> Is a person free to say anything before signing an NDA? </a> </li> <li > <div class="favicon favicon-stats" title="Cross Validated"></div><a href="https://stats.stackexchange.com/questions/661364/what-are-some-real-world-examples-of-statistical-models-where-the-dependent-vari" class="js-gps-track question-hyperlink mb0" data-gps-track="site.switch({ item_type:11, target_site:65 }); posts_hot_network.click({ item_type:2, location:11 })"> What are some real-world examples of statistical models where the dependent variable chronologically occurs before the independent variable? </a> </li> <li > <div class="favicon favicon-tex" title="TeX - LaTeX Stack Exchange"></div><a href="https://tex.stackexchange.com/questions/737265/a-strange-behaviour-of-option-breaklines-of-package-fvextra" class="js-gps-track question-hyperlink mb0" data-gps-track="site.switch({ item_type:11, target_site:85 }); posts_hot_network.click({ item_type:2, location:11 })"> A strange behaviour of option breaklines of package fvextra </a> </li> <li > <div class="favicon favicon-worldbuilding" title="Worldbuilding Stack Exchange"></div><a href="https://worldbuilding.stackexchange.com/questions/264699/how-could-farmers-prevent-unicornification" class="js-gps-track question-hyperlink mb0" data-gps-track="site.switch({ item_type:11, target_site:579 }); posts_hot_network.click({ item_type:2, location:11 })"> How could farmers prevent Unicornification? </a> </li> <li > <div class="favicon favicon-scifi" title="Science Fiction & Fantasy Stack Exchange"></div><a href="https://scifi.stackexchange.com/questions/294956/origin-of-the-word-literjon" class="js-gps-track question-hyperlink mb0" data-gps-track="site.switch({ item_type:11, target_site:186 }); posts_hot_network.click({ item_type:2, location:11 })"> Origin of the word "literjon"? </a> </li> <li class="dno js-hidden"> <div class="favicon favicon-gaming" title="Arqade"></div><a href="https://gaming.stackexchange.com/questions/411177/what-is-the-best-way-to-achieve-enlightenment" class="js-gps-track question-hyperlink mb0" data-gps-track="site.switch({ item_type:11, target_site:41 }); posts_hot_network.click({ item_type:2, location:11 })"> What is the best way to achieve "Enlightenment"? </a> </li> <li class="dno js-hidden"> <div class="favicon favicon-bicycles" title="Bicycles Stack Exchange"></div><a href="https://bicycles.stackexchange.com/questions/96232/why-did-it-take-so-long-for-wide-range-1x-budget-drivetrains-to-appear" class="js-gps-track question-hyperlink mb0" data-gps-track="site.switch({ item_type:11, target_site:126 }); posts_hot_network.click({ item_type:2, location:11 })"> Why did it take so long for wide-range 1x budget drivetrains to appear? </a> </li> <li class="dno js-hidden"> <div class="favicon favicon-diy" title="Home Improvement Stack Exchange"></div><a href="https://diy.stackexchange.com/questions/314684/how-to-fit-a-4-pipe-through-a-3-space" class="js-gps-track question-hyperlink mb0" data-gps-track="site.switch({ item_type:11, target_site:73 }); posts_hot_network.click({ item_type:2, location:11 })"> How to fit a 4" pipe through a 3" space </a> </li> <li class="dno js-hidden"> <div class="favicon favicon-math" title="Mathematics Stack Exchange"></div><a href="https://math.stackexchange.com/questions/5035382/are-these-figures-homeomorphic" class="js-gps-track question-hyperlink mb0" data-gps-track="site.switch({ item_type:11, target_site:69 }); posts_hot_network.click({ item_type:2, location:11 })"> Are these figures homeomorphic? </a> </li> <li class="dno js-hidden"> <div class="favicon favicon-superuser" title="Super User"></div><a href="https://superuser.com/questions/1879825/what-does-444-in-the-language-options-in-7-zip-mean" class="js-gps-track question-hyperlink mb0" data-gps-track="site.switch({ item_type:11, target_site:3 }); posts_hot_network.click({ item_type:2, location:11 })"> What does 444 in the language options in 7-Zip mean? </a> </li> <li class="dno js-hidden"> <div class="favicon favicon-physics" title="Physics Stack Exchange"></div><a href="https://physics.stackexchange.com/questions/842893/the-problem-about-relativity-in-uniform-circular-motion" class="js-gps-track question-hyperlink mb0" data-gps-track="site.switch({ item_type:11, target_site:151 }); posts_hot_network.click({ item_type:2, location:11 })"> The problem about relativity in uniform circular motion </a> </li> <li class="dno js-hidden"> <div class="favicon favicon-english" title="English Language & Usage Stack Exchange"></div><a href="https://english.stackexchange.com/questions/629353/is-there-an-english-proverb-for-ogni-santo-ha-i-suoi-devoti-which-suggests-th" class="js-gps-track question-hyperlink mb0" data-gps-track="site.switch({ item_type:11, target_site:97 }); posts_hot_network.click({ item_type:2, location:11 })"> Is there an English proverb for “Ogni santo ha i suoi devoti”, which suggests that, to different degrees, every person has someone who likes them? </a> </li> <li class="dno js-hidden"> <div class="favicon favicon-hermeneutics" title="Biblical Hermeneutics Stack Exchange"></div><a href="https://hermeneutics.stackexchange.com/questions/101375/when-thomas-said-my-lord-and-my-god-did-he-address-both-the-son-and-the-fath" class="js-gps-track question-hyperlink mb0" data-gps-track="site.switch({ item_type:11, target_site:320 }); posts_hot_network.click({ item_type:2, location:11 })"> When Thomas said, "My Lord and my God", did he address both the Son and the Father? </a> </li> <li class="dno js-hidden"> <div class="favicon favicon-philosophy" title="Philosophy Stack Exchange"></div><a href="https://philosophy.stackexchange.com/questions/122699/is-god-outside-of-reality-for-theist-and-non-theist-naturalists" class="js-gps-track question-hyperlink mb0" data-gps-track="site.switch({ item_type:11, target_site:265 }); posts_hot_network.click({ item_type:2, location:11 })"> Is God outside of reality for theist (and non-theist) naturalists? </a> </li> <li class="dno js-hidden"> <div class="favicon favicon-blender" title="Blender Stack Exchange"></div><a href="https://blender.stackexchange.com/questions/331211/how-to-add-interpolated-children-filler-curves-edge-loops-between-existing-curve" class="js-gps-track question-hyperlink mb0" data-gps-track="site.switch({ item_type:11, target_site:502 }); posts_hot_network.click({ item_type:2, location:11 })"> How to add interpolated children/filler curves/edge loops between existing curves/edge loops on surface with Geometry Nodes? </a> </li> <li class="dno js-hidden"> <div class="favicon favicon-bicycles" title="Bicycles Stack Exchange"></div><a href="https://bicycles.stackexchange.com/questions/96236/most-effective-way-to-make-climbing-gears-easier" class="js-gps-track question-hyperlink mb0" data-gps-track="site.switch({ item_type:11, target_site:126 }); posts_hot_network.click({ item_type:2, location:11 })"> Most effective way to make climbing gears easier </a> </li> <li class="dno js-hidden"> <div class="favicon favicon-law" title="Law Stack Exchange"></div><a href="https://law.stackexchange.com/questions/107396/does-at-will-employment-entitle-you-to-quit-without-giving-any-notice" class="js-gps-track question-hyperlink mb0" data-gps-track="site.switch({ item_type:11, target_site:617 }); posts_hot_network.click({ item_type:2, location:11 })"> Does at-will employment entitle you to quit without giving any notice? </a> </li> <li class="dno js-hidden"> <div class="favicon favicon-worldbuilding" title="Worldbuilding Stack Exchange"></div><a href="https://worldbuilding.stackexchange.com/questions/264708/how-much-carbon-monoxide-would-you-need-to-fill-a-house-for-it-to-combust-when-l" class="js-gps-track question-hyperlink mb0" data-gps-track="site.switch({ item_type:11, target_site:579 }); posts_hot_network.click({ item_type:2, location:11 })"> How much carbon monoxide would you need to fill a house for it to combust when lit? </a> </li> <li class="dno js-hidden"> <div class="favicon favicon-politics" title="Politics Stack Exchange"></div><a href="https://politics.stackexchange.com/questions/90625/could-democrats-take-back-the-house-of-representatives-on-april-1st-with-upcomin" class="js-gps-track question-hyperlink mb0" data-gps-track="site.switch({ item_type:11, target_site:475 }); posts_hot_network.click({ item_type:2, location:11 })"> Could Democrats take back the House of Representatives on April 1st with upcoming special elections? </a> </li> <li class="dno js-hidden"> <div class="favicon favicon-rpg" title="Role-playing Games Stack Exchange"></div><a href="https://rpg.stackexchange.com/questions/214734/faithful-hound-vs-mind-blank" class="js-gps-track question-hyperlink mb0" data-gps-track="site.switch({ item_type:11, target_site:122 }); posts_hot_network.click({ item_type:2, location:11 })"> Faithful hound vs Mind Blank </a> </li> <li class="dno js-hidden"> <div class="favicon favicon-puzzling" title="Puzzling Stack Exchange"></div><a href="https://puzzling.stackexchange.com/questions/130538/eight-circles-in-a-star" class="js-gps-track question-hyperlink mb0" data-gps-track="site.switch({ item_type:11, target_site:559 }); posts_hot_network.click({ item_type:2, location:11 })"> Eight circles in a star </a> </li> <li class="dno js-hidden"> <div class="favicon favicon-crypto" title="Cryptography Stack Exchange"></div><a href="https://crypto.stackexchange.com/questions/114334/xor-sum-versus-additive-sum-for-parallelizable-hash" class="js-gps-track question-hyperlink mb0" data-gps-track="site.switch({ item_type:11, target_site:281 }); posts_hot_network.click({ item_type:2, location:11 })"> XOR sum versus additive sum for parallelizable hash </a> </li> <li class="dno js-hidden"> <div class="favicon favicon-stackoverflow" title="Stack Overflow"></div><a href="https://stackoverflow.com/questions/79442076/is-there-a-way-in-c-to-have-a-scoped-enum-containing-multiple-representations" class="js-gps-track question-hyperlink mb0" data-gps-track="site.switch({ item_type:11, target_site:1 }); posts_hot_network.click({ item_type:2, location:11 })"> Is there a way in C++ to have a scoped enum containing multiple representations? </a> </li> <li class="dno js-hidden"> <div class="favicon favicon-superuser" title="Super User"></div><a href="https://superuser.com/questions/1879973/thousands-of-unknown-accounts-in-file-permissions" class="js-gps-track question-hyperlink mb0" data-gps-track="site.switch({ item_type:11, target_site:3 }); posts_hot_network.click({ item_type:2, location:11 })"> Thousands of unknown accounts in file permissions </a> </li> </ul> <a href="#" class="show-more js-show-more js-gps-track" data-gps-track="posts_hot_network.click({ item_type:3, location:11 })"> more hot questions </a> </div> <div id="feed-link" class="js-feed-link"> <a href="/feeds/question/37503" title="Feed of this question and its answers"> <svg aria-hidden="true" class="fc-orange-400 svg-icon iconRss" width="18" height="18" viewBox="0 0 18 18"><path d="M3 1a2 2 0 0 0-2 2v12c0 1.1.9 2 2 2h12a2 2 0 0 0 2-2V3a2 2 0 0 0-2-2zm0 1.5c6.9 0 12.5 5.6 12.5 12.5H13C13 9.55 8.45 5 3 5zm0 5c4.09 0 7.5 3.41 7.5 7.5H8c0-2.72-2.28-5-5-5zm0 5c1.36 0 2.5 1.14 2.5 2.5H3z"/></svg> Question feed </a> </div> <aside class="s-modal js-feed-link-modal" tabindex="-1" role="dialog" aria-labelledby="feed-modal-title" aria-describedby="feed-modal-description" aria-hidden="true"> <div class="s-modal--dialog js-modal-dialog wmx4" role="document" data-controller="se-draggable"> <h1 class="s-modal--header fw-bold js-first-tabbable" id="feed-modal-title" data-se-draggable-target="handle" tabindex="0"> Subscribe to RSS </h1> <div class="d-flex gs4 gsy fd-column"> <div class="flex--item"> <label class="d-block s-label c-default" for="feed-url"> Question feed <p class="s-description mt2" id="feed-modal-description">To subscribe to this RSS feed, copy and paste this URL into your RSS reader.</p> </label> </div> <div class="d-flex ps-relative"> <input class="s-input" type="text" name="feed-url" id="feed-url" readonly="readonly" value="https://linguistics.stackexchange.com/feeds/question/37503" /> <svg aria-hidden="true" class="s-input-icon fc-orange-400 svg-icon iconRss" width="18" height="18" viewBox="0 0 18 18"><path d="M3 1a2 2 0 0 0-2 2v12c0 1.1.9 2 2 2h12a2 2 0 0 0 2-2V3a2 2 0 0 0-2-2zm0 1.5c6.9 0 12.5 5.6 12.5 12.5H13C13 9.55 8.45 5 3 5zm0 5c4.09 0 7.5 3.41 7.5 7.5H8c0-2.72-2.28-5-5-5zm0 5c1.36 0 2.5 1.14 2.5 2.5H3z"/></svg> </div> </div> <a class="s-modal--close s-btn s-btn__muted js-modal-close js-last-tabbable" href="#" aria-label="Close"> <svg aria-hidden="true" class="svg-icon iconClearSm" width="14" height="14" viewBox="0 0 14 14"><path d="M12 3.41 10.59 2 7 5.59 3.41 2 2 3.41 5.59 7 2 10.59 3.41 12 7 8.41 10.59 12 12 10.59 8.41 7z"/></svg> </a> </div> </aside> </div> </div> <script>StackExchange.ready(function(){$.get('/posts/37503/ivc/f912?prg=4e22c478-0129-40ea-a00a-ede1fd610062');});</script> <noscript><div><img src="/posts/37503/ivc/f912?prg=4e22c478-0129-40ea-a00a-ede1fd610062" class="dno" alt="" width="0" height="0"></div></noscript></div> <script defer src="https://cdn.sstatic.net/Js/questions/show.en.js?v=03feb8d88a8b"></script> <script type="application/json" data-role="module-args" data-module-name="entry-points/questions/show.mod">{"doMarkObsolete":false,"doTimeOnPage":false,"sendAnswerEditorEventsToAEH":false,"sendAnswerEditorEventsToPrizm":false,"questionId":37503,"sendAuthQuestionVisitToAEH":false,"sendAuthQuestionVisitToPrizm":false}</script> </div> </div> <script type="text/javascript"> var cam = cam || { opt: {} }; var clcGamLoaderOptions = cam || { opt: {} }; var opt = clcGamLoaderOptions.opt; opt.refresh = !1; opt.refreshInterval = 0; opt.sf = !1; opt.hb = !1; opt.ll = !0; opt.tlb_position = 0; opt.personalization_consent = !1; opt.targeting_consent = !1; opt.performance_consent = !1; opt.targeting = {Registered:['false'],Reputation:['new'],NumberOfAnswers:['1'],cf_bot_score:'1'}; opt.adReportEnabled = !1; opt.adReportUrl = '/ads/report-ad'; opt.adReportText = 'Report this ad'; opt.adReportFileTypeErrorMessage = 'Please select a PNG or JPG file.'; opt.adReportFileSizeErrorMessage = 'The file must be under 2 MiB.'; opt.adReportErrorText = 'Error uploading ad report.'; opt.adReportThanksText = 'Thanks for your feedback. We’ll review this against our code of conduct and take action if necessary.'; opt.adReportLoginExpiredMessage = 'Your login session has expired, please login and try again.'; opt.adReportLoginErrorMessage = 'An error occurred when loading the report form - please try again'; opt.adReportModalClass = 'js-ad-report'; opt.countryCode = 'SG'; opt.qualtricsSurveyData = '{"isRegistered":"False","repBucket":"new","referrer":"https%3a%2f%2flinguistics.stackexchange.com%2fquestions%2f37503%2fwhat-is-the-reason-for-having-irregular-verbs","accountAge":"0"}'; opt.perRequestGuid = '4e22c478-0129-40ea-a00a-ede1fd610062'; opt.responseHash = 'HtBT4raCg7iWhtodpFR7XnCjtIhXa27pejRJ2KvQPUM='; opt.targeting.TargetingConsent = ['False_Passive']; opt.allowAccountTargetingForThisRequest = !1; const urlParams = new URLSearchParams(window.location.search); if (urlParams.has('dfptestads')) { const dfptestads = urlParams.get('dfptestads'); opt.targeting.DfpTestAds = dfptestads; } </script> <script>;(()=>{"use strict";var __webpack_modules__={23:(e,t,s)=>{s.d(t,{Z7:()=>c,eq:()=>l,kG:()=>d});const n="248424177",o=(a=location.pathname,/^\/tags\//.test(a)||/^\/questions\/tagged\//.test(a)?"tag-pages":/^\/discussions\//.test(a)||/^\/beta\/discussions/.test(a)?"discussions":/^\/$/.test(a)||/^\/home/.test(a)?"home-page":/^\/jobs$/.test(a)||/^\/jobs\//.test(a)?"jobs":"question-pages");var a;let i=location.hostname;const r={slots:{lb:[[728,90]],mlb:[[728,90]],smlb:[[728,90]],bmlb:[[728,90]],sb:e=>"dfp-tsb"===e?[[300,250],[300,600]]:[[300,250]],"tag-sponsorship":[[730,135]],"mobile-below-question":[[320,50],[300,250]],msb:[[300,250],[300,600]],"talent-conversion-tracking":[[1,1]],"site-sponsorship":[[230,60]]},ids:{"dfp-tlb":"lb","dfp-mlb":"mlb","dfp-smlb":"smlb","dfp-bmlb":"bmlb","dfp-tsb":"sb","dfp-isb":"sb","dfp-tag":"tag-sponsorship","dfp-msb":"msb","dfp-sspon":"site-sponsorship","dfp-m-aq":"mobile-below-question"},idsToExcludeFromAdReports:["dfp-sspon"]};function d(){return Object.keys(r.ids)}function l(e){return r.idsToExcludeFromAdReports.indexOf(e)<0}function c(e,t){var s=e.split("_")[0];const a=r.ids[s];let d=r.slots[a];return"function"==typeof d&&(d=d(s)),{path:`/${n}/${t||i}/${a}/${o}`,sizes:d,zone:a}}},865:(e,t,s)=>{function n(e){return"string"==typeof e?document.getElementById(e):e}function o(e){return!!(e=n(e))&&"none"===getComputedStyle(e).display}function a(e){return!o(e)}function i(e){return!!e}function r(e){return/^\s*$/.test(n(e).innerHTML)}function d(e){const{style:t}=e;t.height=t.maxHeight=t.minHeight="auto",t.display="none"}function l(e){const{style:t}=e;t.height=t.maxHeight=t.minHeight="auto",t.display="none",[].forEach.call(e.children,l)}function c(e){const{style:t}=e;t.height=t.maxHeight=t.minHeight="auto",t.removeProperty("display")}function g(e){const t=document.createElement("script");t.src=e,document.body.appendChild(t)}function p(e){return s=e,(t=[]).push=function(e){return s(),delete this.push,this.push(e)},t;var t,s}function h(e){let t="function"==typeof HTMLTemplateElement;var s=document.createElement(t?"template":"div");return e=e.trim(),s.innerHTML=e,t?s.content.firstChild:s.firstChild}s.d(t,{$Z:()=>c,Bv:()=>h,Gx:()=>g,Nj:()=>n,QZ:()=>p,cf:()=>d,pn:()=>a,wo:()=>l,xb:()=>r,xj:()=>o,yb:()=>i})},763:(__unused_webpack_module,__webpack_exports__,__webpack_require__)=>{__webpack_require__.d(__webpack_exports__,{t:()=>AdReports});var _common_helper__WEBPACK_IMPORTED_MODULE_2__=__webpack_require__(865),_console__WEBPACK_IMPORTED_MODULE_1__=__webpack_require__(276),_ad_units__WEBPACK_IMPORTED_MODULE_0__=__webpack_require__(23);class AdReports{constructor(e,t){if(this.googletag=e,this.cam=t,this.allowedFileTypes=["image/png","image/jpg","image/jpeg"],this.ignoreValidation=!1,_console__WEBPACK_IMPORTED_MODULE_1__.cM("Ad reporting init"),this.cam=t,this.callOnButtonClick=e=>this.onButtonClick(e),this.googletag.pubads().addEventListener("slotRenderEnded",e=>this.handleSlotRendered(e)),Array.isArray(t.slotsRenderedEvents)){_console__WEBPACK_IMPORTED_MODULE_1__.cM("Adding report button to "+t.slotsRenderedEvents.length+" events that have transpired");for(var s=0;s<t.slotsRenderedEvents.length;s++)this.handleSlotRendered(t.slotsRenderedEvents[s])}}handleSlotRendered(e){if(e&&e.slot&&!e.isEmpty&&(e.creativeId||e.lineItemId||!e.isEmpty)){var t=e.slot.getSlotElementId();if(t){var s=document.getElementById(t);if(s)if((0,_ad_units__WEBPACK_IMPORTED_MODULE_0__.eq)(t)){var n=s?.closest(".js-zone-container")?.querySelector(".js-report-ad-button-container");n?(n.innerHTML="",n.append(this.createButton(e)),n.style.height="24px",_console__WEBPACK_IMPORTED_MODULE_1__.cM("Added report button to the bottom of "+t)):_console__WEBPACK_IMPORTED_MODULE_1__.cM("Ad report button not found, may be intentional, element: "+t)}else _console__WEBPACK_IMPORTED_MODULE_1__.cM("Not adding report button to the bottom of "+t+": shouldHaveReportButton = false");else _console__WEBPACK_IMPORTED_MODULE_1__.cM("Not adding report button to the bottom of "+t+": resolved invalid adUnit element")}else _console__WEBPACK_IMPORTED_MODULE_1__.cM("Not adding report button to the bottom of element: invalid adUnitElementId")}else _console__WEBPACK_IMPORTED_MODULE_1__.cM("Not adding report button to the bottom of element: invalid SlotRenderEndedEvent")}async onButtonClick(e){e.preventDefault();let t=e.target;const s=t.dataset.modalUrl,n=t.dataset.googleEventData;return await this.loadModal(s,t,n),!1}createButton(e){let t=document.createElement("button");var s=JSON.stringify(e);return t.dataset.googleEventData=s,t.dataset.modalUrl=this.cam.opt.adReportUrl,t.dataset.adUnit=e.slot.getSlotElementId(),t.classList.add("js-report-ad","s-btn","s-btn__link","fs-fine","mt2","float-right"),t.append(document.createTextNode(this.cam.opt.adReportText)),t.removeEventListener("click",this.callOnButtonClick),t.addEventListener("click",this.callOnButtonClick),t}async loadModal(url,$link,googleEventData){try{await window.StackExchange.helpers.loadModal(url,{returnElements:window.$($link)}),this.initForm(googleEventData)}catch(e){var message="",response=e.responseText?eval(`(${e.responseText})`):null;message=response&&response.isLoggedOut?this.cam.opt.adReportLoginExpiredMessage:this.cam.opt.adReportLoginErrorMessage,window.StackExchange.helpers.showToast(message,{type:"danger"})}}removeModal(){window.StackExchange.helpers.closePopups(document.querySelectorAll("."+this.cam.opt.adReportModalClass),"dismiss")}initForm(e,t=!1){this.ignoreValidation=t,this.$form=document.querySelector(".js-ad-report-form"),this.$googleEventData=this.$form.querySelector(".js-json-data"),this.$adReportReasons=this.$form.querySelectorAll(".js-ad-report-reason"),this.$adReportReasonOther=this.$form.querySelector(".js-ad-report-reason-other"),this.$fileUploaderInput=this.$form.querySelector(".js-file-uploader-input"),this.$imageUploader=this.$form.querySelector(".js-image-uploader"),this.$clearImageUpload=this.$form.querySelector(".js-clear-image-upload"),this.$imageUploaderText=this.$form.querySelector(".js-image-uploader-text"),this.$imageUploaderPreview=this.$form.querySelector(".js-image-uploader-preview"),this.$fileErrorMessage=this.$form.querySelector(".js-file-error");const s=this.$form.querySelector(".js-drag-drop-enabled"),n=this.$form.querySelector(".js-drag-drop-disabled");this.$googleEventData.value=e,this.$adReportReasons.forEach((e,t)=>e.addEventListener("change",e=>{this.$adReportReasonOther.classList.toggle("d-none","3"!==e.target.value)})),this.$fileUploaderInput.addEventListener("change",()=>{this.validateFileInput()&&this.updateImagePreview(this.$fileUploaderInput.files)}),this.$clearImageUpload.addEventListener("click",e=>{e.preventDefault(),this.clearImageUpload()});try{this.$fileUploaderInput[0].value="",this.$imageUploader.addEventListener("dragenter dragover dragleave drop",this.preventDefaults),this.$imageUploader.addEventListener("dragenter dragover",this.handleDragStart),this.$imageUploader.addEventListener("dragleave drop",this.handleDragEnd),this.$imageUploader.addEventListener("drop",this.handleDrop)}catch(e){s.classList.add("d-none"),n.classList.remove("d-none")}this.$form.removeEventListener("",this.handleDragEnd),this.$form.addEventListener("submit",async e=>(e.preventDefault(),this.submitForm(),!1))}clearImageUpload(){this.$fileUploaderInput.value="",this.$imageUploaderPreview.setAttribute("src",""),this.$imageUploaderPreview.classList.add("d-none"),this.$clearImageUpload.classList.add("d-none"),this.$imageUploaderText.classList.remove("d-none"),this.$imageUploader.classList.add("p16","ba","bas-dashed","bc-black-100")}preventDefaults(e){e.preventDefault(),e.stopPropagation()}handleDragStart(e){this.$imageUploader.classList.remove("bas-dashed"),this.$imageUploader.classList.add("bas-solid","bc-black-100")}handleDragEnd(e){this.$imageUploader.classList.remove("bas-solid","bc-black-100"),this.$imageUploader.classList.add("bas-dashed")}handleDrop(e){var t=e.originalEvent.dataTransfer.files;FileReader&&t&&1===t.length&&(this.$fileUploaderInput.files=t,this.validateFileInput()&&this.updateImagePreview(t))}setError(e){this.$fileErrorMessage.parentElement.classList.toggle("has-error",e)}updateImagePreview(e){this.$imageUploader.classList.remove("p16","ba","bas-dashed","bc-black-100"),this.$clearImageUpload.classList.remove("d-none"),this.$imageUploaderText.classList.add("d-none");var t=new FileReader;t.onload=e=>{null!=e.target&&(this.$imageUploaderPreview.setAttribute("src",e.target.result),this.$imageUploaderPreview.classList.remove("d-none"))},t.readAsDataURL(e[0])}validateFileInput(){if(this.ignoreValidation)return!0;const e=this.cam.opt.adReportFileTypeErrorMessage,t=this.cam.opt.adReportFileSizeErrorMessage;if(null==this.$fileUploaderInput.files)return!1;var s=this.$fileUploaderInput.files[0];return null==s?(this.setError(!0),!1):this.allowedFileTypes.indexOf(s.type)<0?(this.$fileErrorMessage.textContent=e,this.$fileErrorMessage.classList.remove("d-none"),this.setError(!0),!1):s.size>2097152?(this.$fileErrorMessage.textContent=t,this.$fileErrorMessage.classList.remove("d-none"),this.setError(!0),!1):(this.$fileErrorMessage.classList.add("d-none"),this.setError(!1),!0)}async gatherDiagnosticInfo(){return{BrowserVersion:await this.getBrowserVersion()}}getElementSource(e){return e.outerHTML}getNestedIFrameElement(e){var t=e.querySelector("iframe");return t.contentDocument?t.contentDocument.documentElement:t.contentWindow.document.documentElement}async getBrowserVersion(){return await navigator.userAgentData.getHighEntropyValues(["fullVersionList"]).then(e=>JSON.stringify(e.fullVersionList))}async submitForm(){if(!this.validateFileInput())return!1;this.$form.querySelector("[type=submit]").setAttribute("disabled","true");var e=JSON.parse(this.$googleEventData.value||"{}");e.Reason=parseInt(this.$form.querySelector(".js-ad-report-reason:checked").value,10),e.Description=this.$adReportReasonOther.value,this.$googleEventData.value=JSON.stringify(e);var t=new FormData(this.$form);if("1"===t.get("shareDiagnosticInfo")){var s=await this.gatherDiagnosticInfo();Object.keys(s).forEach(e=>t.append(e,s[e]))}try{const e=await window.fetch(this.$form.getAttribute("action"),{method:this.$form.getAttribute("method"),body:t,cache:"no-cache"}),s=e.headers.get("content-type")||"",o=await e.text();if(!e.ok)throw new Error("response not valid");if(0===s.indexOf("text/html")){var n=(0,_common_helper__WEBPACK_IMPORTED_MODULE_2__.Bv)(o);const e=n?n.querySelector(".js-modal-content"):null;if(_console__WEBPACK_IMPORTED_MODULE_1__.cM("$popupContent"),_console__WEBPACK_IMPORTED_MODULE_1__.cM(e),!e)throw new Error(`Could not find .js-modal-content in response from ${this.$form.getAttribute("action")}`);document.querySelector(".js-modal-content").replaceWith(e)}else window.StackExchange.helpers.showToast(this.cam.opt.adReportThanksText,{type:"success"}),this.removeModal()}catch(e){window.StackExchange.helpers.showToast(this.cam.opt.adReportErrorText,{type:"danger"})}finally{let e=this.$form.querySelector("[type=submit]");e&&e.removeAttribute("disabled")}}}},276:(e,t,s)=>{function n(...e){}function o(...e){}s.d(t,{cM:()=>n,vU:()=>o})}},__webpack_module_cache__={};function __webpack_require__(e){var t=__webpack_module_cache__[e];if(void 0!==t)return t.exports;var s=__webpack_module_cache__[e]={exports:{}};return __webpack_modules__[e](s,s.exports,__webpack_require__),s.exports}__webpack_require__.d=(e,t)=>{for(var s in t)__webpack_require__.o(t,s)&&!__webpack_require__.o(e,s)&&Object.defineProperty(e,s,{enumerable:!0,get:t[s]})},__webpack_require__.o=(e,t)=>Object.prototype.hasOwnProperty.call(e,t);var __webpack_exports__={};(()=>{var e=__webpack_require__(276),t=(e=>(e[e.Above=0]="Above",e[e.Below=1]="Below",e))(t||{});const s=Object.assign({},{"lib":"https://clc.stackoverflow.com/Content/bundles/js/gam_loader_script.bundle.741.5deb8dc1a671bfb1646b.js?v=7178a92a336c","style":null,"u":null,"wa":true,"kt":2000,"tto":true,"h":"clc.stackoverflow.com","allowed":"^(((talent\\.)?stackoverflow)|(blog\\.codinghorror)|(.*\\.googlesyndication)|(serverfault|askubuntu|superuser)|([^\\.]+\\.stackexchange))\\.com$","wv":true,"al":false,"abd":true,"cpa_liid":[5882654614],"cpa_cid":[138377597667],"dp":false,"tgt_to":1000,"tgt_u":"https://clc.stackoverflow.com/get-user-acct-tgt","tgt_e":true,"tgt_p":100,"dv_enabled":false,"upo":""});var n=__webpack_require__(23),o=__webpack_require__(865),a=__webpack_require__(763);class i{constructor(t,s){this.googletag=t,this.interval=s,e.cM("Ad refresh init. interval: "+s),this.googletag.pubads().addEventListener("impressionViewable",e=>this.onImpressionViewable(e)),e.cM("done enabling ad refresh")}onImpressionViewable(t){var s=t.slot;e.cM("ad refresh - slot "+s.getSlotElementId()+" is viewable, initializing refresh"),this.scheduleRefresh(s)}scheduleRefresh(e){setTimeout(()=>this.refreshAdSlot(e),1e3*this.interval)}static refreshMyAd(t,s){let n=t.pubads().getSlots().find(e=>e.getSlotElementId()===s);n&&(e.cM("refreshMyAd - refreshing ad slot "+s),t.pubads().refresh([n]))}static removeMyAd(t,s){let n=t.pubads().getSlots().find(e=>e.getSlotElementId()===s);n&&(e.cM("removeMyAd - destroying ad slot "+s),t.destroySlots([n]))}refreshAdSlot(t){var s=t.getSlotElementId();this.isElementVisibleInBrowser(s)?(e.cM("refreshing ad slot "+s),googletag.pubads().refresh([t])):(e.cM("refresh skipped this time; ad slot not viewable:"+s),this.scheduleRefresh(t))}isElementVisibleInBrowser(e){var t=document.getElementById(e);if(null!==t){var s=t.getBoundingClientRect();if(s.top>=0&&s.left>=0&&s.bottom<=(window.innerHeight||document.documentElement.clientHeight)&&s.right<=(window.innerWidth||document.documentElement.clientWidth))return!0}return!1}}var r=(e=>(e.Off="Off",e.PreSurvey="PreSurvey",e.Collect="Collect",e.PostSurvey="PostSurvey",e))(r||{});class d{constructor(e,t){this.lineItemImpressions=[],this.surveysIdsCompleted=[],this.lineItemImpressions=e,this.surveysIdsCompleted=t}addImpression(e,t){let s={brandId:e,lineItemId:t,timestamp:new Date};this.lineItemImpressions.push(s)}addBrandSurveyCompleted(e){-1===this.surveysIdsCompleted.indexOf(e)&&this.surveysIdsCompleted.push(e)}getTotalBrandImpressions(){let e=new Map;for(let t of this.lineItemImpressions)if(e.has(t.brandId)){let s=e.get(t.brandId);e.set(t.brandId,s+1)}else e.set(t.brandId,1);return e}getBrandLineItemImpressions(e){let t={};for(let s of this.lineItemImpressions)if(s.brandId==e)if(void 0!==t[s.lineItemId]){let e=t[s.lineItemId];t[s.lineItemId]=e+1}else t[s.lineItemId]=1;return t}}class l{constructor(){this.surveyEngagementLocalStorageKey="clc-survey-engagement"}getBrandSurveyEngagement(){let e=localStorage.getItem(this.surveyEngagementLocalStorageKey);if(null===e)return new d([],[]);let t=JSON.parse(e);return new d(t.lineItemImpressions,t.surveysIdsCompleted)}saveBrandSurveyEngagement(e){let t=JSON.stringify(e);localStorage.setItem(this.surveyEngagementLocalStorageKey,t)}}class c{constructor(){this.surveyRepository=new l}getBrandSurveyEngagement(){return this.surveyRepository.getBrandSurveyEngagement()}recordImpression(e,t){let s=this.getBrandSurveyEngagement();s.addImpression(e,t),this.surveyRepository.saveBrandSurveyEngagement(s)}recordBrandSurveyCompleted(e){let t=this.getBrandSurveyEngagement();t.addBrandSurveyCompleted(e),this.surveyRepository.saveBrandSurveyEngagement(t)}}class g{constructor(t,s){this.googletag=t,this.brandSettings=s,this.brandSlotMap=new Map,this.brandSurveyEngagementService=new c,e.cM("Brand Survey init: "+JSON.stringify(s)),void 0!==s?(this.googletag.pubads().addEventListener("slotRenderEnded",e=>this.handleSlotRendered(e)),this.googletag.pubads().addEventListener("impressionViewable",e=>this.onImpressionViewable(e)),e.cM("done enabling Brand Survey")):e.cM("Brand Survey init: brandSettings is undefined, not initializing")}handleSlotRendered(t){e.cM("Brand Survey - slot rendered - slot:"+JSON.stringify(t.slot.getSlotElementId())+" lineItem: "+t.lineItemId);let s=this.findItemWithId(t.lineItemId);if(null===s||s.mode!==r.Collect)this.brandSlotMap.delete(t.slot.getSlotElementId());else{let e={brandId:s.brandId,lineItemId:t.lineItemId};this.brandSlotMap.set(t.slot.getSlotElementId(),e)}}onImpressionViewable(t){let s=t.slot;if(e.cM("ad - Brand Survey - impression viewable. Details: "+JSON.stringify(s.getSlotElementId())),e.cM("ad - Brand Survey - slot "+s.getSlotElementId()+" is viewable"),this.brandSlotMap.has(s.getSlotElementId())){let t=this.brandSlotMap.get(s.getSlotElementId());e.cM("Brand Survey - brand "+t.brandId+" is viewable"),this.recordImpression(this.brandSlotMap.get(s.getSlotElementId()))}}recordImpression(t){e.cM("ad - Brand Survey - recording impression for brand "+t.brandId),this.brandSurveyEngagementService.recordImpression(t.brandId,t.lineItemId)}findItemWithId(t){return e.cM("brand settings: "+JSON.stringify(this.brandSettings)),this.brandSettings.find(e=>e.lineItemIds.includes(t))||null}}const p="response-brand-survey-submit|",h="request-brand-survey-metadata|",m="record-metric-on-server|",u="request-dsp-tags",f="response-dsp-tags|";class _{static refreshAdIfBrandSurveyIsDuplicated(e,t,s){if(this.alreadyCompletedThisBrandSurvey(t)){var n=document.getElementById(s).closest(".js-zone-container");i.removeMyAd(e,s),n&&n.remove()}}static alreadyCompletedThisBrandSurvey(e){return(new c).getBrandSurveyEngagement().surveysIdsCompleted.includes(e)}}window.cam=new class{constructor(t=null){if(this.gptImported=!1,this.slotsRenderedEvents=[],this.collapsed={},e.cM("constructor"),this.clc_options=s,window.clcGamLoaderOptions)Object.assign(this,window.clcGamLoaderOptions);else if(void 0===this.opt){let e=window.opt;e&&(this.opt=e)}}init(){if(e.cM("init"),void 0===this.opt)throw new Error("opt not set, required by GAM Loader");e.cM("init brand survey service"),this.getUserMetaPromise=this.getUserMeta(),e.cM("setup message handler"),window.addEventListener("message",e=>{this.onmessage(e)})}handleSlotRenderedNoAdReport(){if(googletag.pubads().addEventListener("slotRenderEnded",e=>this.applyExtraMarginBottom(e)),Array.isArray(this.slotsRenderedEvents))for(var e=0;e<this.slotsRenderedEvents.length;e++)this.applyExtraMarginBottom(this.slotsRenderedEvents[e])}onmessage(t){let s="omni";if(t.data&&("string"==typeof t.data||t.data instanceof String))if(0===t.data.indexOf("get-omni-")){e.cM("Recevied get-omni message, sending back omni");var n=t.source,a=this.opt.omni,i="string"==typeof a?a:"";n.postMessage([s,i,this.opt.perRequestGuid].join("|"),"*")}else if(0===t.data.indexOf("collapse-")){e.cM("Recevied collapse message, collapse ad iframe"),e.cM(t);for(var r=t.source.window,d=document.getElementsByTagName("IFRAME"),l=0;l<d.length;l++){var g=d[l];if(g.contentWindow==r)return void(0,o.wo)(g.parentElement.parentElement.parentElement)}}else if(0===t.data.indexOf("resize|")){e.cM("Recevied resize message, resize ad iframe"),e.cM(t);let s=this._getFrameByEvent(t),n=t.data.indexOf("|")+1,o=t.data.slice(n),a=parseFloat(o)+.5;e.cM("New iframe height "+a),s.height=a.toString(),s.parentElement.style.height=a.toString()+"px"}else if(0===t.data.indexOf("getmarkup|")){let s=t.data.indexOf("|")+1,n=t.data.slice(s);e.cM("Recevied get markup message: "+n);let o=this._getFrameByEvent(t).closest(".everyonelovesstackoverflow");const a=document.createElement("script");a.dataset.adZoneId=o.id,a.src=n,document.body.appendChild(a)}else if(0===t.data.indexOf("window-location|")){let s=t.data.indexOf("|")+1,n=t.data.slice(s);e.cM("Recevied window location message: "+n),n.startsWith("/")||(n="/"+n),window.open(window.location.protocol+"//"+window.location.host+n,"_blank")}else if(0===t.data.indexOf("request-brand-survey-submit|")){let s=t.data.split("|"),n=s[1],o=s[2],a=s[3],i=JSON.parse(a);e.cM(n),e.cM(o),e.cM(a),e.cM("Received brand survey "+n+" response message: "+o);var v=new FormData;for(var b in i)v.append(b,i[b]);let r=this._getFrameByEvent(t);if(_.alreadyCompletedThisBrandSurvey(+n))return e.cM("Already completed this brand survey. Not submitting duplicate to server."),void r.contentWindow.postMessage("response-brand-survey-submit-duplicate|","*");e.cM("Send the brand survey to the server"),fetch(o,{method:"POST",body:v}).then(e=>e.json()).then(e=>r.contentWindow.postMessage({messageType:p},"*")).catch(e=>r.contentWindow.postMessage({messageType:p},"*"))}else if(0===t.data.indexOf("brand-survey-completed-store|")){let s=t.data.split("|"),n=(s[1],s[2]);if(e.cM("Received brand survey completed store message for survey ID "+n),_.alreadyCompletedThisBrandSurvey(+n))return void e.cM("Already completed this brand survey. Not recording duplicate locally.");e.cM("Record brand survey completion locally"),(new c).recordBrandSurveyCompleted(+n)}else if(0===t.data.indexOf(h)){let s=t.data.split("|"),n=s[1],o=s[2];e.cM("Received message: "+h+" with Brand Survey ID "+o);let a=(new c).getBrandSurveyEngagement().getBrandLineItemImpressions(+n),i=JSON.stringify(a),r=this._getFrameByEvent(t);e.cM("sending impression data: "+i),r.contentWindow.postMessage("response-brand-survey-metadata|"+this.opt.responseHash+"|"+this.opt.perRequestGuid+"|"+i+"|"+this.opt.countryCode+"|"+this.opt.qualtricsSurveyData,"*")}else if(0===t.data.indexOf("refresh-if-duplicate-brand-survey|")){let e=t.data.split("|")[1],s=this.getSlotElementIdByEvent(t);_.refreshAdIfBrandSurveyIsDuplicated(googletag,+e,s)}else if(0===t.data.indexOf(m)){e.cM("Received message: "+m+" with args: "+t.data);let s=t.data.split("|"),n=s[1],o=s[2],a=s[3],i=s[4],r=new FormData;r.append("brandSurveyId",a.toString()),r.append("responseHash",this.opt.responseHash),r.append("perRequestGuid",this.opt.perRequestGuid),r.append("questionNumber",n.toString()),r.append("metricType",i.toString()),fetch(o,{method:"POST",body:r}).then(e=>e.ok).catch(t=>{e.cM("SendMetricToServer: Error sending metric to server: "+t)})}else if(0===t.data.indexOf(u)){e.cM("Received message: "+u+" with args: "+t.data);let s=this._getFrameByEvent(t);if(!this.opt.targeting["so-tag"])return void s.contentWindow.postMessage(f,"*");const n=this.opt.targeting["so-tag"].join(",");e.cM("sending targeting tags: "+n),s.contentWindow.postMessage(f+n,"*")}else e.cM("Received unhandled message")}getSlotElementIdByEvent(e){let t=this._getFrameByEvent(e),s=t.parentElement?.parentElement?.id;return s||""}_getFrameByEvent(e){return Array.from(document.getElementsByTagName("iframe")).filter(t=>t.contentWindow===e.source)[0]}classifyZoneIds(e){const t=e.map(o.Nj).filter(o.yb);return{eligible:t.filter(o.xb).filter(o.pn),ineligible:t.filter(o.xj)}}applyExtraMarginBottom(t){if(t&&t.slot&&!t.isEmpty&&(t.creativeId||t.lineItemId||!t.isEmpty)){var s=t.slot.getSlotElementId();if(s){var o=document.getElementById(s);if(o)if((0,n.eq)(s)){var a=o?.closest(".js-zone-container");a.style.marginBottom="24px",e.cM("Applied extra margin to the bottom of "+s)}else e.cM("Not applying extra margin to the bottom of "+s+": shouldHaveReportButton = false");else e.cM("Not applying extra margin to the bottom of "+s+": resolved invalid adUnit element")}else e.cM("Not applying extra margin to the bottom of element: invalid adUnitElementId")}else e.cM("Not applying extra margin to the bottom of element: invalid SlotRenderEndedEvent")}async load(s=(0,n.kG)()){const r=this.opt.tlb_position===t.Above?["dfp-mlb","dfp-smlb"]:["dfp-mlb","dfp-smlb","dfp-tlb"];if(!this.isGptReady())return e.cM("Initializing..."),this.initGpt(),void googletag.cmd.push(()=>this.load(s));this.opt.adReportEnabled?(e.cM("Ad reporting enabled"),this.adReports=new a.t(googletag,this)):(e.cM("Ad reporting not enabled"),this.handleSlotRenderedNoAdReport()),this.opt.refresh?(e.cM("Ad refresh enabled"),this.adRefresh=new i(googletag,this.opt.refreshInterval)):e.cM("Ad refresh not enabled"),this.opt.brandSurveyEnabled&&(e.cM("Brand Survey enabled"),this.brandSurvey=new g(googletag,this.opt.brandSurveySettings)),e.cM("Attempting to load ads into ids: ",s);const{eligible:d,ineligible:l}=this.classifyZoneIds(s);if(this.initDebugPanel(googletag,d.concat(l)),d.forEach(e=>(0,o.cf)(e)),l.forEach(o.wo),0===d.length)return void e.cM("Found no ad ids on page");e.cM("Eligible ids:",d),this.opt.abd&&this.appendAdblockDetector();var c=googletag.pubads().getSlots();if(c){var p=c.filter(e=>s.indexOf(e.getSlotElementId())>=0);googletag.destroySlots(p)}this.opt.sf&&(googletag.pubads().setForceSafeFrame(!0),googletag.pubads().setSafeFrameConfig({allowOverlayExpansion:!0,allowPushExpansion:!0,sandbox:!0})),e.cM("Targeting consent: Checking...");let h=!1,m=!1;void 0!==this.opt.targeting_consent&&(m=!0,e.cM("Targeting consent: Parameter set"),e.cM("Targeting consent: Consent given? ",this.opt.targeting_consent),h=this.opt.targeting_consent),void 0!==this.opt.personalization_consent&&(e.cM("Personalization consent: Parameter set"),e.cM("Personalization consent: Consent given? ",this.opt.personalization_consent),h=h&&this.opt.personalization_consent),h=h&&m,this.setPrivacySettings(h),this.opt.ll||googletag.pubads().enableSingleRequest(),cam.sreEvent||(googletag.pubads().addEventListener("slotRenderEnded",e=>this.onSlotRendered(e)),cam.sreEvent=!0),await this.setTargeting();var u=d.filter(e=>!this.opt.ll||r.indexOf(e.id)<0),f=d.filter(e=>!!this.opt.ll&&r.indexOf(e.id)>=0);e.cM("Up front ids:",u),e.cM("Lazy loaded ids:",f),u.forEach(t=>{e.cM(`Defining ad for element ${t.id}`),this.defineSlot(t.id,googletag),t.setAttribute("data-dfp-zone","true")}),googletag.enableServices(),u.forEach(t=>{e.cM(`Displaying ad for element ${t.id}`),this.clc_options.dv_enabled?window.onDvtagReady(function(){googletag.display(t.id)}):googletag.cmd.push(()=>googletag.display(t.id))}),this.opt.ll&&(e.cM("Enabling lazy loading for GAM"),googletag.pubads().enableLazyLoad({fetchMarginPercent:0,renderMarginPercent:0}),e.cM("Setting up lazy loaded ad units"),f.forEach(t=>{e.cM(`Lazy loading - Defining Slot ${t.id}`),this.defineSlot(t.id,googletag)}),f.forEach(t=>{e.cM(`Lazy loading - Displaying ad for element ${t.id}`),this.clc_options.dv_enabled?window.onDvtagReady(function(){googletag.display(t.id)}):googletag.cmd.push(()=>googletag.display(t.id))}))}setPrivacySettings(e){e||googletag.pubads().setPrivacySettings({nonPersonalizedAds:!0})}async setTargeting(){if(!googletag)throw new Error("googletag not defined");let t=this.opt.targeting;if(!t)throw new Error("Targeting not defined (is "+typeof t+")");Object.keys(t).forEach(s=>{e.cM(`-> targeting - ${s}: ${t[s]}`),googletag.pubads().setTargeting(s,t[s])});let s=!1;if(void 0!==this.opt.targeting_consent&&(s=this.opt.targeting_consent),s){let t=(new c).getBrandSurveyEngagement();if(t.getTotalBrandImpressions().forEach((t,s)=>{e.cM(`-> targeting - BrandImpressions: ${s}: ${t}`),googletag.pubads().setTargeting("brand_"+s.toString()+"_impressions",t.toString())}),t.surveysIdsCompleted.forEach(t=>{e.cM(`-> targeting - SurveysTaken: ${t}`),googletag.pubads().setTargeting("survey_"+t+"_taken","true")}),this.clc_options.tgt_e&&this.getUserMetaPromise){let t=await this.getUserMetaPromise;t&&t.tgt_acct?(e.cM("-> targeting - User Account: "+t.tgt_acct),googletag.pubads().setTargeting("user-acct",t.tgt_acct.company_name),googletag.pubads().setTargeting("user_acct_top",t.tgt_acct.company_name),googletag.pubads().setTargeting("user_industry",t.tgt_acct.industry),googletag.pubads().setTargeting("user_employee_count",t.tgt_acct.employee_range)):e.cM("-> targeting - User Account: Not Found"),t&&Object.prototype.hasOwnProperty.call(t,"is_high_rep_earner")?(e.cM("-> targeting - High Rep Earner: "+t.is_high_rep_earner),googletag.pubads().setTargeting("IsHighRepEarner",t.is_high_rep_earner?"true":"false")):e.cM("-> targeting - High Rep Earner: not found")}if(localStorage){e.cM('Checking local storage for "jobs-last-clicked" key.');let t=localStorage.getItem("jobs-last-clicked")?"true":"false";e.cM(`-> targeting - jobs_clicked: ${t}`),googletag.pubads().setTargeting("jobs_clicked",t)}}}appendAdblockDetector(){const e=document.createElement("div");e.className="adsbox",e.id="clc-abd",e.style.position="absolute",e.style.pointerEvents="none",e.innerHTML=" ",document.body.appendChild(e)}onSlotRendered(s){try{const i=s.slot.getSlotElementId();let r=[];i||r.push("id=0");const d=document.getElementById(i);if(i&&!d&&r.push("el=0"),0!==r.length)return void this.stalled(r.join("&"));const{path:l,sizes:c,zone:g}=(0,n.Z7)(i,this.clc_options.upo);if(this.collapsed[g]&&s.isEmpty)return e.cM(`No line item for the element #${d.id}... collapsing.`),void(0,o.wo)(d);if(this.slotsRenderedEvents.push(s),s.lineItemId||s.creativeId||!s.isEmpty){e.cM(`Rendered ad for element #${d.id} [line item #${s.lineItemId}]`),e.cM(s);var a=d.parentElement;if(a.classList.contains("js-zone-container")){switch((0,o.cf)(a),i){case"dfp-tlb":this.opt.tlb_position===t.Above?a.classList.add("mb8"):a.classList.add("mt16");break;case"dfp-tag":a.classList.add("mb8");break;case"dfp-msb":a.classList.add("mt16");break;case"dfp-mlb":case"dfp-smlb":case"dfp-bmlb":a.classList.add("my8");break;case"dfp-isb":a.classList.add("mt24");break;case"dfp-m-aq":a.classList.add("my12"),a.classList.add("mx-auto")}(0,o.$Z)(a),(0,o.$Z)(d)}else e.cM(`No ad for element #${d.id}, collapsing`),e.cM(s),(0,o.wo)(d)}}catch(t){e.cM("Exception thrown onSlotRendered"),e.cM(t),this.stalled("e=1")}}stalled(e){(new Image).src=`https://${this.clc_options.h}/stalled.gif?${e}`}defineSlot(t,s){"dfp-isb"===t&&(e.cM("-> targeting - Sidebar: Inline"),s.pubads().setTargeting("Sidebar",["Inline"])),"dfp-tsb"===t&&(e.cM("-> targeting - Sidebar: Right"),s.pubads().setTargeting("Sidebar",["Right"]));const{path:o,sizes:a,zone:i}=(0,n.Z7)(t,this.clc_options.upo);e.cM(`Defining slot for ${t}: ${o}, sizes: ${JSON.stringify(a)}`),s.defineSlot(o,a,t).addService(s.pubads())}importGptLibrary(){this.gptImported||(this.gptImported=!0,void 0===this.opt.targeting_consent||this.opt.targeting_consent?(0,o.Gx)("https://securepubads.g.doubleclick.net/tag/js/gpt.js"):(0,o.Gx)("https://pagead2.googlesyndication.com/tag/js/gpt.js"))}importDvLibrary(){this.clc_options.dv_enabled&&(e.cM("Adding DoubleVerify library"),(0,o.Gx)("https://pub.doubleverify.com/dvtag/21569774/DV1289064/pub.js"),e.cM("Adding DoubleVerify onDvtagReady handler"),window.onDvtagReady=function(t,s=750){e.cM("DoubleVerify onDvtagReady called"),window.dvtag=window.dvtag||{},dvtag.cmd=dvtag.cmd||[];const n={callback:t,timeout:s,timestamp:(new Date).getTime()};dvtag.cmd.push(function(){dvtag.queueAdRequest(n)}),setTimeout(function(){const e=n.callback;n.callback=null,e&&e()},s)})}isGptReady(){return"undefined"!=typeof googletag&&!!googletag.apiReady}initGpt(){"undefined"==typeof googletag&&(window.googletag={cmd:(0,o.QZ)(()=>{this.importGptLibrary(),this.importDvLibrary()})})}getUserMeta(){if(this.opt.allowAccountTargetingForThisRequest&&this.clc_options.tgt_e&&this.clc_options.tgt_p>0){if(e.cM("Targeting enabled."),this.clc_options.tgt_p<100){e.cM("Targeting rate limit enabled. Rolling the dice...");const t=Math.floor(100*Math.random())+1;if(e.cM("Rolled "+t+" and the max is "+this.clc_options.tgt_p),t>this.clc_options.tgt_p)return void e.cM("Will not request targeting.")}return e.cM("Will request targeting."),function(e,t,s,n){if(t){const t=new Headers;return t.append("Accept","application/json"),async function(e,t={},s=5e3){if("number"!=typeof s&&null!=s&&!1!==s){if("string"!=typeof s)throw new Error("fetchWithTimeout: timeout must be a number");if(s=parseInt(s),isNaN(s))throw new Error("fetchWithTimeout: timeout must be a number (or string that can be parsed to a number)")}const n=new AbortController,{signal:o}=n,a=fetch(e,{...t,signal:o}),i=setTimeout(()=>n.abort(),s);try{const e=await a;return clearTimeout(i),e}catch(e){throw clearTimeout(i),e}}(s+"?"+new URLSearchParams({omni:e}),{method:"GET",mode:"cors",headers:t},n).then(e=>e.json())}return Promise.reject("No consent")}(this.opt.omni,this.opt.targeting_consent,this.clc_options.tgt_u,this.clc_options.tgt_to).catch(t=>{e.vU("Error fetching user account targeting"),e.vU(t)})}e.cM("Targeting disabled. Will not request account targeting data.")}initDebugPanel(t,s){e.cM("initDebugPanel"),e.cM("Not showing debug panel.")}},window.clcGamLoaderOptions&&(cam.init(),cam.load())})()})();</script> <footer id="footer" class="site-footer js-footer" role="contentinfo"> <div class="site-footer--container"> <nav class="site-footer--nav" aria-label="Footer"> <div class="site-footer--col"> <h5 class="-title"><a href="/">Linguistics</a></h5> <ul class="-list js-primary-footer-links"> <li><a class="js-gps-track -link" data-gps-track="footer.click({ location: 2, link: 2 })" href="/tour">Tour</a></li> <li><a href="/help" class="js-gps-track -link" data-gps-track="footer.click({ location: 2, link: 3 })">Help</a></li> <li><a class="js-gps-track -link" data-gps-track="footer.click({ location: 2, link: 5 })" href="https://chat.stackexchange.com?tab=site&host=linguistics.stackexchange.com">Chat</a></li> <li><a class="js-gps-track -link" data-gps-track="footer.click({ location: 2, link: 13 })" href="/contact">Contact</a></li> <li><a class="js-gps-track -link" data-gps-track="footer.click({ location: 2, link: 14 })" href="https://linguistics.meta.stackexchange.com">Feedback</a></li> </ul> </div> <div class="site-footer--col"> <h5 class="-title"><a class="js-gps-track" data-gps-track="footer.click({ location: 2, link: 1 })" href="https://stackoverflow.co/">Company</a></h5> <ul class="-list"> <li><a href="https://stackoverflow.com" class="js-gps-track -link" data-gps-track="footer.click({ location: 2, link: 15})">Stack Overflow</a></li> <li><a href="https://stackoverflow.co/teams/" class="js-gps-track -link" data-gps-track="footer.click({ location: 2, link: 29 })">Teams</a></li> <li><a href="https://stackoverflow.co/advertising/" class="js-gps-track -link" data-gps-track="footer.click({ location: 2, link: 21 })">Advertising</a></li> <li><a href="https://stackoverflow.co/advertising/employer-branding/" class="js-gps-track -link" data-gps-track="footer.click({ location: 2, link: 20 })">Talent</a></li> <li><a class="js-gps-track -link" data-gps-track="footer.click({ location: 2, link: 1 })" href="https://stackoverflow.co/">About</a></li> <li><a class="js-gps-track -link" data-gps-track="footer.click({ location: 2, link: 27 })" href="https://stackoverflow.co/company/press/">Press</a></li> <li><a class="js-gps-track -link" data-gps-track="footer.click({ location: 2, link: 7 })" href="https://stackoverflow.com/legal">Legal</a></li> <li><a class="js-gps-track -link" data-gps-track="footer.click({ location: 2, link: 8 })" href="https://stackoverflow.com/legal/privacy-policy">Privacy Policy</a></li> <li><a class="js-gps-track -link" data-gps-track="footer.click({ location: 2, link: 37 })" href="https://stackoverflow.com/legal/terms-of-service/public">Terms of Service</a></li> <li id="consent-footer-link"><button type="button" data-controller="cookie-settings" data-action="click->cookie-settings#toggle" class="s-btn s-btn__link py4 js-gps-track -link" data-gps-track="footer.click({ location: 2, link: 38 })" data-consent-popup-loader="footer">Cookie Settings</button></li> <li><a class="js-gps-track -link" data-gps-track="footer.click({ location: 2, link: 39 })" href="https://stackoverflow.com/legal/cookie-policy">Cookie Policy</a></li> </ul> </div> <div class="site-footer--col site-footer--categories-nav"> <div> <h5 class="-title"><a href="https://stackexchange.com" data-gps-track="footer.click({ location: 2, link: 30 })">Stack Exchange Network</a></h5> <ul class="-list"> <li> <a href="https://stackexchange.com/sites#technology" class="-link js-gps-track" data-gps-track="footer.click({ location: 2, link: 24 })"> Technology </a> </li> <li> <a href="https://stackexchange.com/sites#culturerecreation" class="-link js-gps-track" data-gps-track="footer.click({ location: 2, link: 24 })"> Culture & recreation </a> </li> <li> <a href="https://stackexchange.com/sites#lifearts" class="-link js-gps-track" data-gps-track="footer.click({ location: 2, link: 24 })"> Life & arts </a> </li> <li> <a href="https://stackexchange.com/sites#science" class="-link js-gps-track" data-gps-track="footer.click({ location: 2, link: 24 })"> Science </a> </li> <li> <a href="https://stackexchange.com/sites#professional" class="-link js-gps-track" data-gps-track="footer.click({ location: 2, link: 24 })"> Professional </a> </li> <li> <a href="https://stackexchange.com/sites#business" class="-link js-gps-track" data-gps-track="footer.click({ location: 2, link: 24 })"> Business </a> </li> <li class="mt16 md:mt0"> <a href="https://api.stackexchange.com/" class="-link js-gps-track" data-gps-track="footer.click({ location: 2, link: 24 })"> API </a> </li> <li> <a href="https://data.stackexchange.com/" class="-link js-gps-track" data-gps-track="footer.click({ location: 2, link: 24 })"> Data </a> </li> </ul> </div> </div> </nav> <div class="site-footer--copyright fs-fine md:mt24"> <ul class="-list -social md:mb8"> <li><a class="js-gps-track -link" data-gps-track="footer.click({ location: 2, link:4 })" href="https://stackoverflow.blog?blb=1">Blog</a></li> <li><a href="https://www.facebook.com/officialstackoverflow/" class="-link js-gps-track" data-gps-track="footer.click({ location: 2, link: 31 })">Facebook</a></li> <li><a href="https://twitter.com/stackoverflow" class="-link js-gps-track" data-gps-track="footer.click({ location: 2, link: 32 })">Twitter</a></li> <li><a href="https://linkedin.com/company/stack-overflow" class="-link js-gps-track" data-gps-track="footer.click({ location: 2, link: 33 })">LinkedIn</a></li> <li><a href="https://www.instagram.com/thestackoverflow" class="-link js-gps-track" data-gps-track="footer.click({ location: 2, link: 36 })">Instagram</a></li> </ul> <p class="md:mb0"> <span>Site design / logo © 2025 Stack Exchange Inc; </span> <span>user contributions licensed under </span> <a class="-link s-link td-underline" href="https://stackoverflow.com/help/licensing">CC BY-SA</a> <span>. </span> <span id="svnrev">rev 2025.2.14.22661</span> </p> </div> </div> </footer> <!-- Google tag (gtag.js) --> <script async src="https://www.googletagmanager.com/gtag/js?id=G-S812YQPLT2"></script> <script> window.dataLayer = window.dataLayer || []; function gtag() { dataLayer.push(arguments); } </script> <script> StackExchange.ready(function() { var ga3Settings = { autoLink: ["stackoverflow.blog","info.stackoverflowsolutions.com","stackoverflowsolutions.com"], sendTitles: true, tracker: window.ga, trackingCodes: [ 'UA-108242619-5' ], checkDimension: 'dimension42' }; var customGA4Dimensions = {}; customGA4Dimensions["requestid"] = "4e22c478-0129-40ea-a00a-ede1fd610062"; customGA4Dimensions["routename"] = "Questions/Show"; customGA4Dimensions["post_id"] = "37503"; customGA4Dimensions["tags"] = "|verbs|irregularity|"; var ga4Settings = { tracker: gtag, trackingCodes: [ 'G-S812YQPLT2' ], consentsToPerformanceCookies: "denied", consentsToTargetingCookies: "denied", eventParameters: customGA4Dimensions, checkForAdBlock: true, sendTitles: true, trackClicks: false, }; StackExchange.ga.init({ GA3: ga3Settings, GA4: ga4Settings }); StackExchange.ga.setDimension('dimension2', '|verbs|irregularity|'); StackExchange.ga.setDimension('dimension3', 'Questions/Show'); StackExchange.ga.setDimension('dimension7', "1739733042.816612183"); StackExchange.ga.trackPageView(); }); </script> <script src="https://cdn.cookielaw.org/scripttemplates/otSDKStub.js" charset="UTF-8" data-document-language="true" data-domain-script="cb0f3c87-b769-4e66-bbaa-377f9194216d"></script> <script defer src="https://cdn.sstatic.net/Js/modules/cookie-consent.en.js?v=36bebc18e04f"></script> <script>(function(){function c(){var b=a.contentDocument||a.contentWindow.document;if(b){var d=b.createElement('script');d.innerHTML="window.__CF$cv$params={r:'912fd6dd9b684056',t:'MTczOTczMzA0My4wMDAwMDA='};var a=document.createElement('script');a.nonce='';a.src='/cdn-cgi/challenge-platform/scripts/jsd/main.js';document.getElementsByTagName('head')[0].appendChild(a);";b.getElementsByTagName('head')[0].appendChild(d)}}if(document.body){var a=document.createElement('iframe');a.height=1;a.width=1;a.style.position='absolute';a.style.top=0;a.style.left=0;a.style.border='none';a.style.visibility='hidden';document.body.appendChild(a);if('loading'!==document.readyState)c();else if(window.addEventListener)document.addEventListener('DOMContentLoaded',c);else{var e=document.onreadystatechange||function(){};document.onreadystatechange=function(b){e(b);'loading'!==document.readyState&&(document.onreadystatechange=e,c())}}}})();</script></body> </html>